Trauma Flashcards

1
Q

<p>Damage to which portion of frontal sinus would most likely involve the naso-frontal ducts? A. Posterior wall B. Medial portion of floor C. Anterior wall D. Lateral portion of floor</p>

A

<p>Answer: B Rationale: Damage to the medial portion of the frontal sinus most often involves damage to the naso- frontal ducts. This is an important area to evaluate intraoperatively to evaluate the patency of the naso-frontal ducts. Other areas mentioned do not involve the NFD's. Posterior wall fractures involve the brain, and the lateral aspect of the sinus can involve the orbital roof. Reference: Fonseca. Oral and Maxillofacial Trauma. Evaluation and management of frontal sinus injuries, p 721-736, 2005. Peterson. Principles of Oral and Maxillofacial Surgery. Management of frontal sinus and naso-orbitalethmoid complex fractures, 491-508, 2004.</p>

How well did you know this?
1
Not at all
2
3
4
5
Perfectly
2
Q

<p>During a Gillies approach to access the zygomatic arch, the plane of dissection is between which two anatomic layers?
A. Superficial and deep layers of deep temporalis fascia
B. Temporoparietal fascia and superficial layer of deep temporalis fascia
C. Subcutaneous fat layer and temporoparietal fascia
D. Deep layer of deep temporalis fascia and temporalis muscle</p>

A

<p>Answer: D
Rationale:
The Gillies approach to reduce a zygomatic arch fracture utilizes a dissection between the deep layer of the deep temporalis fascia and the temporalis muscle. The plane of dissection is sub-fascial, but supra-muscular. Dissection in the temporoparietal fascia may damage the facial nerve and will not reach the zygomatic arch. Since the two layers of the deep temporal fascia split to envelop the arch, dissection must be maintained between the muscle and deep layer of the deep temporal fascia in order to place the elevator deep to the arch.
American Board of Oral and Maxillofacial Surgery
144
2008 Oral and Maxillofacial Surgery Self Assessment Tool (OMSSAT)
Reference:
Fonseca. Oral and Maxillofacial. Trauma. Evaluation and management of frontal sinus injuries, p 721-736, 2005.
Peterson. Principles of Oral and Maxillofacial Surgery. Management of frontal sinus and naso-orbitalethmoid complex fractures, 491-508, 2004.</p>

How well did you know this?
1
Not at all
2
3
4
5
Perfectly
3
Q

<p>The marginal mandibular nerve is:
A. anterior to the facial artery, but not the vein, in 100% of cases.
B. anterior to the facial vein, but not the artery, in 100% of cases.
C. below the inferior border of the mandible in 19% of the cases where the nerve is posterior to the facial vessels.
D. below the inferior border of the mandible in 5% of the cases where the nerve is anterior to the facial vessels.</p>

A

<p>Answer: C
Rationale:
The classic 1961 study by Dingman and Grabb showed the relationship of the marginal mandibular branch of the facial nerve in relation to the facial vessels. In 81% of the cases, the nerve was superior to the inferior border of the mandible posterior to the vessels. In 19% of the cases, the nerve ran up to 1 centimeter inferior to the inferior border of the mandible posterior to the vessels, and in 100% of cases, when the nerve was anterior to the vessels, the marginal mandibular branch ran above the inferior border of the mandible. Since the marginal mandibular branch runs from posterior to anterior, it cross the vertically running facial artery and vein and therefore always is found to run anterior to both facial vessels.
Reference:
Dingman RO, Grabb WC. Surgical anatomy of the mandibular ramus of the facial nerve based on the dissection of 100 facial halves. Plastic Reconstrutive Surgery Vol 29, 1962.
Ziarah HA, Atkinson ME. The surgical anatomy of the mandibular distribution of the facial nerve. Br J Oral Surg. 1981 Sep; 19(3):159-70.</p>

How well did you know this?
1
Not at all
2
3
4
5
Perfectly
4
Q
<p>The Keen technique is utilized to reduce fracture of what structure?
A. Mandibular condyle
B. Medial canthal tendon
C. Coronoid process
D. Zygomatic arch</p>
A

<p>Answer: D
Rationale:
Keen and Carmedy-Baxon approaches are the two classic methods of approaching and reducing a zygomatic arch trans-orally. Buccal sulcus (vestibular) is the Keen approach, while the lateral coronoid approach (along the ascending ramus) is known as the Carmedy- Baxon. These two approaches allow reduction of a fractured arch The Keen approach allows access to the infraorbital rim and nasomaxillary region as well while the Carmedy- Baxon approach is somewhat more limited in exposure.
Reference:
Quinn JH. Lateral coronoid approach for intraoral reduction of fractures of the zygomatic arch. J Oral Surg 35; 1977.
Courtney DJ. Upper buccal sulcus approach to management of fractures of the zygomatic complex: a retrospective study of 50 cases. Br J Oral Maxillofac Surg, 37, 1999.</p>

How well did you know this?
1
Not at all
2
3
4
5
Perfectly
5
Q
<p>What is the most important bony suture for future growth when treating a naso-septal fracture in the pediatric population?
A. Nasomaxillary
B. Nasofrontal
C. Septovomerine
D. Frontoethmoidal
</p>
A

<p>Answer: C
Rationale:
The septovomerine junction is considered a growth center in the pediatric nose. Therefore, adequate reduction of this region is imperative to decrease risk of premature ossification which can lead to future growth disturbance.
Reference:
Haug RH.. Maxillofacial injuries in the pediatric patient. Oral Surg, Oral Medicine, Oral Path, 90, 126, 2000.
Rock WP, Brain DJ. The effects of nasal trauma during childhood upon growth of the nose and midface. Br J Orthod 10: 38, 1983.</p>

How well did you know this?
1
Not at all
2
3
4
5
Perfectly
6
Q

<p>What is the most important factor in re-establishment of the vertical facial height in the management of pan-facial fractures?
A. Reduction of mandibular condyle fractures
B. Reduction of fronto-zygomatic sutures
C. Establishing proper zygomatic projection
D. Establishing proper dental occlusion</p>

A

<p>Answer: A
Rationale:
There are several key landmarks when treating pan-facial injuries. While re-establishing proper occlusion is a key component, the posterior facial height and ramus/condyle units must be aligned properly in order to gain vertical height of the face. This is accomplished by proper reduction of mandibular condyles after proper occlusion is established. If teeth are not present, then anatomic reduction of the mandible and/or maxilla must be performed in order to establish proper vertical height. Once proper occlusion and vertical ramus height are established, alignment of vertical and horizontal facial bony buttresses is accomplished.
Reference:
Peterson. Principles of Oral and Maxillofacial Surgery. Management of panfacial fractures, 547-562, 2004.
Markowitz BL, Manson PN. Panfacial fractures: organization of treatment. Clin Plast Surg 16; 105, 1989.</p>

How well did you know this?
1
Not at all
2
3
4
5
Perfectly
7
Q

<p>Between which two anatomic layers is the safest plane of dissection during the initial elevation of a coronal flap?
A. Subcutaneous fat and galea
B. Pericranium and galea
C. Subcutaneous fat and temporoparietal fascia
D. Galeal fat pad and temporal fat pad</p>

A

<p>Answer: B
Rationale:
The safest plane of dissection in a coronal flap elevation is the subgaleal plane located between the galea and the pericranium. At this level, the frontal branch is above (superior) the level of the dissection and thus is less susceptible to injury.
Reference:
Frodel JL, Marentette LJ. The coronal approach. Anatomic and technical considerations and morbidity. Arch Otolaryngol Head Neck Surg. 1993 Feb; 119(2):140.
Liebman EP, Webster RC, Berger AS, et al. The frontalis nerve in the temporal brow lift. Arch Otolaryngol 1982 Apr; 108(4):232-5.
</p>

How well did you know this?
1
Not at all
2
3
4
5
Perfectly
8
Q
<p>What is the most frequently fractured area of the edentulous mandible?
A. Condyle
B. Subcondyle
C. Symphysis
D. Body
</p>
A

<p>Answer: D
Rationale:
43.5% of all fractures of the edentulous mandible occur in the body region. This is compared to only 33% in the dentate patient. The body fracture is the most frequently fractured segment of the edentulous mandible.
Reference:
Bruce RA, Strachan DS. Fractures of the edentulous mandible: the Chalmers j. Lyons Academy stud. J Oral Surg, 34:9073, 1976.
Peterson 2nd edition, Principles of Oral and Maxillofacial Surgery, Principles of management of mandibular fractures, 401-434, 2004
Fonseca, Oral and Maxillofacial Trauma, Mandibular fractures, P 479-522, 2005</p>

How well did you know this?
1
Not at all
2
3
4
5
Perfectly
9
Q
<p>What is the mostly commonly used endoscope (offset degree of angulation and diameter) for repair of mandibular condyle fractures?
A. 30 degrees, 2mm
B. 0 degrees, 2 mm
C. 30 degrees, 4mm
D. 0 degrees, 4 mm</p>
A

<p>Answer: C
Rationale:
Although endoscopically assisted condyle repairs are not as commonly performed as the more traditional methods, the most commonly used endoscope is a 30', 4 mm endoscope. This technique requires making a Rison incision and inserting an endoscope while reducing the fracture. Application of fixation can be done directly or via a trocar system.
Reference:
Martin M, Lee C. Endoscopic mandibular condyle fracture repair. Atlas Oral Maxillofac Surg Clin N Am, 11, 2003.
Troulis MJ. Endoscopic open reduction and internal rigid fixation of subcondylar fractures. J Oral Maxillofac Surg 62, 2004.</p>

How well did you know this?
1
Not at all
2
3
4
5
Perfectly
10
Q
<p>Which of the following differentiates between superior orbital fissure syndrome and orbital apex syndrome?
A. Ophthalmoplegia
B. Loss of vision
C. Ptosis of the upper lid
D. Anesthesia of the forehead
</p>
A

<p>Answer: B
Rationale:
Loss of vision is the defining clinical sign which differentiates superior orbital fissure syndrome from orbital apex syndrome. All other signs and symptoms listed are seen in both conditions. Ophthalmoplegia occurs due to involvement of cranial nerves III, IV, VI. Anesthesia occurs due to the involvement of V1. Ptosis occurs due to loss of sympathetic tone to the Muller's muscle; since the terminal sympathetic fibers travel with the ophthalmic branch of the trigeminal nerve (V1.)
Reference:
Peter ward Booth (ed); Maxillofacial trauma; Periorbital and intraorbital trauma and orbital reconstruction, P 205-222, Churchhill Livingstone; 2003
Eo S, Kim JY, Azari K. Temporary orbital apex syndrome after repair of orbital wall fracture. Plast Reconstr Surg. 2005 Oct; 116(5):85e-89e</p>

How well did you know this?
1
Not at all
2
3
4
5
Perfectly
11
Q
<p>Which of the following can cause binocular diplopia?
A. Retinal detachment
B. Lens dislocation
C. Corneal scarring
D. Alteration in globe position
</p>
A

<p>Answer: D
Rationale:
Binocular diplopia is more common than monocular diplopia; however both can result from traumatic insult to the globe. Retinal detachment, lens dislocation, cataracts, and corneal scarring are causes of monocular diplopia. Globe position changes can lead to binocular diplopia.
Reference:
Peter ward Booth (ed); Maxillofacial trauma Periorbital and intraorbital trauma and orbital reconstruction, P 205-222, Churchhill Livingstone; 2003
Michael Miloro (ed); Peterson's principles of Oral and Maxillofacial Surgery 2nd edition, Orbital and ocular trauma, P 463-490, BC Decker Inc, 2004</p>

How well did you know this?
1
Not at all
2
3
4
5
Perfectly
12
Q

<p>What is the mean distance of the optic canal from the posterior ethmoidal foramen? A. 1mm B. 5mm C. 10 mm D. 12mm</p>

<p></p>

<div></div>

A

<p>Answer: B Rationale: When measuring from the anterior lacrimal crest the mean distance of the anterior and posterior ethmiodal foramen are 24 and 36mm respectively, the optic canal is a mean of 5mm posterior to the posterior ethmiodal foramen (or a mean of 42mm from the anterior lacrimal crest). Reference: Michael Miloro (ed); Peterson's principles of Oral and Maxillofacial Surgery 2nd edition, BC Decker Inc, Orbital and ocular trauma, P 463-490,2004, Ray Fonseca (ed); Oral and Maxillofacial Surgery, volume 3; Orbital Trauma, p 205-244, WB Saunders, 2000 <img></img></p>

How well did you know this?
1
Not at all
2
3
4
5
Perfectly
13
Q
What is the greatest width of an upper eye lid defect that can be repaired by primary closure without compromising function?
A. 10%
B. 15%
C. 20%
D. 25%
A

Answer: D
Rationale:
Eyelid injuries involving less then 25% can be closed primarily, those which are 25-50% can be repaired with local tissue advancement. Those greater then a 50% defect will require a flap or skin graft which replaces both the anterior and posterior lamellae.
Reference:
Michael Miloro (ed); Peterson’s principles of Oral and Maxillofacial Surgery 2nd edition, BC Decker Inc, Orbital and ocular trauma, P 463-490, 2004
Ray Fonseca (ed); Oral and Maxillofacial Surgery, volume 3; Orbital Trauma, p 205- 244WB Saunders, 2000

How well did you know this?
1
Not at all
2
3
4
5
Perfectly
14
Q
A 30 year-old male has bilateral lower leg deformities following a motor vehicle collision. He is anxious, with the follwing vital signs: BP = 130/100, pulse = 110, respiratory rate = 28What is his class of blood loss?
A. Class I
B. Class II
C. Class III
D. Class IV
A

Answer: B
Rationale:
Class II Hemorrhage represents volume loss of 750 to 1500ml of blood. Clinical symptoms include tachycardia, tachypnea and a decrease in pulse pressure. This decrease in pulse pressure is primarily related to a rise in the diastolic component due to an increase in circulating catecholamines which increase the vascular tone and resistance. Systolic pressure changes minimally in early hemorrhagic shock.
Reference:
Advanced Trauma Life Support Student Manual. Shock, 87-107, American College of Surgeons. Sixth Edition. 1997.
Miloro, M. et al, Peterson’s Principles of Oral and Maxillofacial Surgery, Second Edition, Initial management of the trauma patient, p 327-356, BC Decker 2004.

How well did you know this?
1
Not at all
2
3
4
5
Perfectly
15
Q
A 25 year-old male presents unconscious following a fall. Neurological evaluation shows that he withdraws from pain, there is no verbal response, and there is no eye opening. What is his Glasgow Coma Score?
A. 4 
B. 5 
C. 6 
D. 7
A

Answer: C
Rationale:
Glasgow Coma Scale is based on three variables: best motor response, best verbal response and eye opening. (GCS = M+V+E) Scores range from 3 to 15. In this case withdrawing from pain represents 4, no verbal response represents 1 and no eye opening represents 1. GCS = (M4+V1+E1) = 6

Reference:
Advanced Trauma Life Support Student Manual. Shock, 87-107, American College of Surgeons. Sixth Edition. 1997.
American Board of Oral and Maxillofacial Surgery
158
2008 Oral and Maxillofacial Surgery Self Assessment Tool (OMSSAT)
Miloro, M. et al, Peterson’s Principles of Oral and Maxillofacial Surgery, Second Edition,
Initial management of the trauma patient, p 327-356, BC Decker 2004. .

How well did you know this?
1
Not at all
2
3
4
5
Perfectly
16
Q
After a severe head trauma, the intracranial pressure of 45 year-old male is 30mm Hg. Other vital signs include pulse 90, respiratory rate 20, and blood pressure 130/85. What is the cerebral perfusion pressure of this patient?
A. 50
B. 60
C. 70
D. 80
A

Answer: C
Rationale:
Cerebral blood flow, maintained by autoregulation, depends on cerebral perfusion pressure and intracranial pressure. Cerebral perfusion pressure is the mean arterial blood pressure minus intracranial pressure. (CPP = MAP ñ ICP). Normal CPP in an adult is >50 mm Hg.
One method for the calculation of the mean arterial pressure is diastolic pressure times 2 plus systolic pressure all divided by three.
{MAP = [(2 x Diastolic Pressure) + Systolic Pressure] / 3}
=[(2 x 85) + 130] / 3
=100
CPP = 100 ñ 30 CPP = 70
Reference:
Advanced Trauma Life Support Student Manual. Shock, 87-107American College of Surgeons. Sixth Edition. 1997.
Way, L et al. Current Surgical Diagnosis and Treatment. Fluid and electrolyte management, 129-142, Eleventh Edition. McGraw Hill. 2003.

How well did you know this?
1
Not at all
2
3
4
5
Perfectly
17
Q
A 2 year-old male presents with second degree burns of his entire head. What percentage of his body surface area is involved?
A. 4.5%
B. 9%
C. 10%
D. 18%
A

Answer: D
Rationale:
The ìRule of Ninesî is a useful and practical guide to determine the extent of the burn and fluid resuscitation. The infant’s head represents a larger proportion of the surface area. The percentage of total body surface of the infant’s head is twice that of the normal adult (which is 9%).
Reference:
Advanced Trauma Life Support Student Manual. Pediatric trauma, p 289-311, American College of Surgeons. Sixth Edition. 1997.
Way, L et al. Current Surgical Diagnosis and Treatment. Fluid and electrolyte management, 129-14 Eleventh Edition. McGraw Hill. 2003.


How well did you know this?
1
Not at all
2
3
4
5
Perfectly
18
Q
Which of the following is a component of Beckís triad?
A. Wide pulse pressure
B. Decreased central venous pressure
C. Increased systolic arterial pressure
D. Muffled heart tones
A

Answer: D
Rationale:
Beck’s triad for diagnosis of cardiac tamponade consists of venous pressure elevation, decline in arterial pressure and muffled heart tones. Wide pulse pressure is seen in shock.
Reference:
Advanced Trauma Life Support Student Manual. Thoracic trauma, p 125-141, American College of Surgeons. Sixth Edition. 1997.
Miloro, M. et al, Peterson’s Principles of Oral and Maxillofacial Surgery, Second Edition, Initial management of the trauma patient, p 327-356BC Decker 2004.

How well did you know this?
1
Not at all
2
3
4
5
Perfectly
19
Q
What is the most common midfacial fracture in the pediatric population?
A. Orbital roof
B. Orbital floor
C. Zygomatico-maxillary
D. Nose
A

Answer: D
Rationale:
The nose is a prominent structure in children and the nasal bones are fragile. Nasal fractures are the most common midfacial skeleton injury in children. The low incidence of midfacial fractures in children can also be explained by the elasticity of the child’s facial bones, the retrusive position of the maxilla, nose and infraorbital rims and the anatomic protection afforded by the cranium.
Reference:
Kaban LB, et. al. Pediatric Oral and Maxillofacial Surgery. Ch. 24 Facial Trauma I: Midfacial Fractures, Baumann A, et. al. p.426.
2 Posnick JC. Craniofacial and Maxillofacial Surgery in Children and Young Adults. Ch 30 Primary Craniomaxillofacial Fracture Management, Posnick JC. p.720.

How well did you know this?
1
Not at all
2
3
4
5
Perfectly
20
Q
The clinical sign differentiating superior orbital fissure syndrome from orbital apex syndrome is:
A. absence of superior palpebral fold
B. proptosis
C. dilated and fixed pupil
D. decreased visual acuity
A

ANSWER: D
RATIONALE:
Symptoms of superior orbital fissure syndrome include:
1. Pupillary dilation via alteration in cranial nerve III function in it’s innervation of the
pupillary constrictors.
2. Paresis of cranial nerves III, IV, and IV causing ophthalmoplegia.
3. Cranial nerve III involvement causes paresis of the levator palpebrae superiorus muscle,
leading to ptosis and loss of the superior palpebral fold.
4. Neurosensory disturbance to the first division of cranial nerve V with hypesthesia of the
supraorbital and supratrochlear nerves and loss of the corneal reflex.
5. Proptosis from engorgement of the ophthalmic vein and lymphatics.
The orbital apex syndrome includes all of the above plus optic nerve involvement, leading to changes in visual acuity.

How well did you know this?
1
Not at all
2
3
4
5
Perfectly
21
Q
Post auricular ecchymosis in cases of high velocity trauma is usually indicative of:
A. fracture of the vertex of the skull
B. mandibular fracture
C. basilar skull fracture
D. LeFort III fracture
A

ANSWER: C
RATIONALE:
A fracture of the skull base results in the extrusion of blood subperiosteally. This hematoma or ecchymosis may be exhibited at the thin skinned mastoid region (post- auricular), as hemotympanum, as bilateral periorbital ecchymosis, or as a posterior pharyngeal ecchymosis or hematoma.

How well did you know this?
1
Not at all
2
3
4
5
Perfectly
22
Q
Alignment of which of the following is the most reliable for proper reduction of the zygomaticomaxillary complex fracture?
A. Frontozygomatic suture
B. Sphenozygomatic suture
C. Infraorbital rim
D. Medial orbital rim
A

ANSWER: B
RATIONALE:
The sphenozygomatic suture area has been previously analyzed and shown to be an area for confirmation of alignment of the zygomatic arch and the zygomatic complex (ZMC). This has also been shown to key point for fixation thru biomechanical studies.
The sphenozygomatic suture is a broad area along the greater wing of the sphenoid and can be approached along the internal aspect of the lateral orbit. Even in severe midface fractures the greater wing of the sphenoid is intact thus acting as a key landmark for proper reduction of the ZMC fracture.
Reduction of the frontozygomatic suture or the infraorbital rim alone can result in errors due to the small surface area. The medial orbit is generally not involved in a ZMC fracture.

How well did you know this?
1
Not at all
2
3
4
5
Perfectly
23
Q

Which of the following is the least cosmetic surgical approach for an adolescent with an orbital floor fracture?
A. Subciliary incision
B. Post septal transconjunctival incision
C. Infraorbital incision
D. Preseptal transconjunctival incision.

A

ANSWER: C
RATIONALE:
Although the infraorbital incision provides direct and excellent exposure of the orbital rim and floor with a low incidence of complications, it frequently produces a noticeable scar. In younger people, this scar increases in size with growth. The subciliary incision is more cosmetic. Whether pre- or post-septal, the transconjunctival incisions do not involve the skin and are cosmetically hidden.

How well did you know this?
1
Not at all
2
3
4
5
Perfectly
24
Q

The first step in the general order of treatment of panfacial fractures is:
A. Establish soft and hard tissue reduction
B. Expose all fracture sites
C. Alleviate soft tissue entrapments
D. Apply internal fixation

A

ANSWER: B
RATIONALE:
The first issue in the order of treatment of panfacial fractures is to ascertain the sites and conditions of the disrupted anatomical structures. This can only be accomplished by exposure of the entire injured facial skeleton. Soft tissue entrapments are next alleviated, the osseous fractures are then reduced, and rigid fixation is applied followed by soft tissue approximation.

How well did you know this?
1
Not at all
2
3
4
5
Perfectly
25
Q

Acute dacryocystitis following trauma is treated by all of the following except:
A. warm compresses
B. intubation of the canaliculi and injection of dye
C. systemic or topical nasal decongestants
D. incision and drainage

A

ANSWER: B
RATIONALE:
Dilation, intubation and dye injection are diagnostic, not therapeutic measures. Moreover, these maneuvers should not be attempted in the face of an acute dacryocystitis. Incision and drainage of the lacrimal sac, administration of medicaments (systemic or topical decongestants,) or palliative care(warm compresses) are acceptable treatment modalities.

How well did you know this?
1
Not at all
2
3
4
5
Perfectly
26
Q

Epiphora can be caused by all of the following except:
A. Telecanthus with rounding of the medial canthus
B. Entropion of the lower lid
C. Ectropion of the lower lid
D. A soft tissue laceration of the lateral aspect of the upper eyelid

A

ANSWER: D
RATIONALE:
Ectropion and entropion can affect the contact of the inferior lacrimal punctum with the tear fluid decreasing lacrimal fluid flow through the punctum and leadin to epiphora. Traumatic telecanthus can also lead to alterations in tear flow and drainage in the medial aspect of the inferior palpebral area and decrease lacrimal drainage through the inferior canilculus. The codnition affect lacrimal fluid drainage but not lacrimal fluid delivery to the palpebral fissure. Although a laceration through the laterial aspect of the upper eyelid can disrupt tear flow from the lacrimal gland, such a decrease in tear production would not lead to epiphora.

How well did you know this?
1
Not at all
2
3
4
5
Perfectly
27
Q
Disruption of the nasolacrimal apparatus with subsequent epiphora occurs most commonly after which facial fracture:
A. Nasal
B. LeFort III
C. Nasoethmoidal
D. Zygomaticomaxillary
A

ANSWER: C
RATIONALE:
The incidence of nasolacrimal disruption is 0.2% following nasal fracture, 3-4% following midface fractures, 17-21 % following naso-ethmoidal fractures, almost non existent following zygomatic-maxillary fractures. The location of the zygoma is so far removed from the lacrimal apparatus so as to make it a concomitant injury.

How well did you know this?
1
Not at all
2
3
4
5
Perfectly
28
Q

Confirmation of a CSF leak following a fontal sinus fracture is best done with which of the following imaging studies?
A. A high resolution computed tomography cisternogram after administration of intrathecal fluorescein
B. A facial series of radiographs to include a Caldwell and lateral view
C. A non contrast computed tomography study of the brain
D. Magnetic resonance imaging of the base of the skull

A

ANSWER: A
RATIONALE:
Plane radiography is incapable of confirming a CSF leak. While magnetic resonance is helpful with soft tissues, without dye, this diagnostic aid is useless. A similar rationale exists for non- contrast CT. An intrathecal injection of dye, and confirmation of the dye at distant sites is diagnostic.

How well did you know this?
1
Not at all
2
3
4
5
Perfectly
29
Q
The most likely diagnosis in a patient with painful proptosis, progressive visual loss, restricted extraocular movement, and increased intraocular pressure following surgery to reduce a zygomatic fracture is:
A. Horner’s syndrome
B. Movement of an alloplastic implant
C. Injury to the infraorbital nerve
D. Retrobulbar hematoma
A

ANSWER: D
RATIONALE:
Movement of an alloplastic implant is generally asymptomatic. Injury to the infraorbital nerve produces anesthesia or paresthesia over it’s cutaneous distribution (the lower eyelid area). Horner’s syndrome, caused by a disruption in the sympathetic innervation to the upsilateral maxillofacial region, is characterized by: a constricted pupil (by unopposed parasympathetic constriction), ptosis (by loss of smpathetic inneration to Mueller’s muscle), and anhidrosis (by interruption of sympathetic innervation to cutaneous sweat glands). The symptoms described are most consistent with retrobulbar hematoma, and require prompt diagnosis and intervention.

How well did you know this?
1
Not at all
2
3
4
5
Perfectly
30
Q
Suspension wires utilized to stabilize a LeFort I fracture resists forces in which direction?
A. Superior
B. Inferior
C. Anterior
D. Posterior
A

ANSWER: B
RATIONALE:
The recent development and improvements in miniaturized bone plate systems has greatly enhanced treatment of midface fractures and diminished but not obviated the need for wire suspension with direct wiring techniques. While wires may provide minor resistance to deformation in an anterior and posterior direction, they offer no resistance superiorly. The best answer is that the resist deformation in an inferior direction and thereby resist facial elongation.

How well did you know this?
1
Not at all
2
3
4
5
Perfectly
31
Q

When evaluating visual acuity in the orbital trauma patient:
A. The pupils should be dilated.
B. Eyeglasses should not be worn by the patient, even if available.
C. Viewing through a pinhole can compensate for some refractory errors.
D. Topical tetracaine can aid acuity evaluation.

A

ANSWER: C
RATIONALE:
Dilation may mask signs and symptoms of neurologic injury. Pupillary dilation does not aid in a visual acuity examination but is utilized to fully visualize the retina, vessels and the optic disc. Pre-existing visual acuity defecits (such as myopia and presbyopia) can mimic traumatic visual acuity loss; and therefore the use of prescription eyeglasses can facilitate the distinction of pretaumatic from traumatic visual defecits. Topical tetracaine is a local anesthetic and is of no value in the evaluation of visual acuity. If pretraumatic myopia is prsent, acuity evaluation while looking through a pinhole can substitute somewhat for corrective lenses if such lenses are not available.

How well did you know this?
1
Not at all
2
3
4
5
Perfectly
32
Q

Which of the following is true when treating eyelid lacerations:
A. Eyelid and ocular mobility should be evaluated before injecting local anesthetic.
B. Fat herniation is not an indication of orbital septum violation.
C. Fat herniation is not an indication of possible globe penetrating injury.
D. Iridocyclitis describes an irregularly shaped pupil which “points” away from the area of
globe injury.

A

ANSWER: A
RATIONALE:
Iridocyclitis is a traumatic anisocoria and many times points towards the injury. Fat herniation occurs with aging and of itself is not necessarily indicative of traumatic septum violation. Herniated fat without lid laceration is therefore of no consequence; however, fat herniation through a lid laceration indicates septum violation and mandates the need for careful evaluation for penetrating globe injury. Penetrating globe injury is diagnosed by visualization of the globe surface. The lid should be examined prior to the administration of local anesthesia because edema and local anesthesia may limit motility.

How well did you know this?
1
Not at all
2
3
4
5
Perfectly
33
Q

Which of the following is true when treating injuries of the external ear:
A. Cartilage should never be sutured, thereby avoiding necrosis.
B. Loose or macerated skin should be extensively débrided.
C. Cartilage lacerations should be sutured with conventional, interrupted chronic gut sutures
to encourage overlapping.
D. Cartilage should be sutured with slowly resorbable figure-of-eight sutures.

A

ANSWER: D
RATIONALE:
Cartilagenous lacerations should be approximated to reconstruct anatomy and prevent chronic chondritic inflammation. Interrupted sutures may promote cartilage margin overlap; the use of figure of eight sutures prevents overlap of lacerated cartilage margins. Cartilage has a limited vascurlar supply, originating from overlying soft tissue; therfore extensive debridement of overlying soft tissue should be discourage.

How well did you know this?
1
Not at all
2
3
4
5
Perfectly
34
Q

When the medial canthal ligament is attached to a bony segment in naso-orbito-ethmoidal(NOE) fracture repair the transcanthal wire is best placed:
A. after all soft tissue injuries have been addressed.
B. anterior to the original insertion of the canthal ligament.
C. posterior and inferior to the original insertion.
D. posterior and superior to the original insertion.

A

ANSWER: D
RATIONALE:
The purpose of the trans-canthal wire is to secure the canthal ligament and boney segment in the pretraumatic position. Pull of the soft tissues displaces the bone and canthal ligament in an anterior and inferior direction. Therefore a wire placed posterior and superior to the original insertion provides a vector whose resistance to displacement is most ideal and provides the best alignment.

How well did you know this?
1
Not at all
2
3
4
5
Perfectly
35
Q

An 18-year-old man is stabbed to the left upper chest. You record a blood pressure of 75/60. He is gasping for air, breath sounds are diminished on the left, and his trachea is deviated to the right. The initial treatment should be:
A. Perform tracheal intubation.
B. Obtain a chest x-ray to verify the pneumothorax.
C. Place a chest tube between the anterior and midaxillary line in the fifth intercostal space.
D. Perform needle decompression of the left chest.

A

ANSWER: D
RATIONALE:
This is a classic tension pneumothorax and treatment is a clinical diagnosis with immediate needle decompression of the second intercostal space, midclavicular line The time involved in waiting for a chest x-ray might prove lethal. Insertion of a chest tube in a controlled fashion to facillitate lung re-expansion normally follows needle deompression of the tension pneumothroax. Endotracheal intumbation with positive pressure ventilation often worsen a tension pneumothrorax, but may be indicated for other types of chest injuries.

How well did you know this?
1
Not at all
2
3
4
5
Perfectly
36
Q

A 21-year-old female is an unrestrained driver involved in a MVA. She suffers a scalp laceration and is noted to have lost 1000mL of blood at the scene. You would expect her vital signs to be consistent with:
A. Pulse rate >100, normal systolic blood pressure, decreased pulse pressure, respiratory rate of 20-30, urinary output of 20-30mL/hr.
B. Pulse rate 30mL/hr.
C. Pulse rate >120, decreased systolic blood pressure, decreased pulse pressure, respiratory rate of 30-40, urinary output of 5-15mL/hr.
D. Pulse rate >140, decreased systolic blood pressure, decreased pulse pressure, respiratory rate of >35, urinary output that’s negligible.

A

ANSWER: A
RATIONALE:
These findings are consistent with a Class II hemorrhage, 750-1500ml, The vitals signs or such a blood loss are consistent with those in response A. Response D reflects the vital signs of a Type IV blood loss, Response C a Type III and Response B a Type I.

How well did you know this?
1
Not at all
2
3
4
5
Perfectly
37
Q
A 65-year-old man fell down the stairs. Upon examination of him, you notice that he opens his eyes to speech, localizes pain, and mutters inappropriate words. You assess his Glasgow coma scale (GCS) to be:
A. 13
B. 11
C. 9 
D. 7
A

ANSWER: B
RATIONALE:
According to the Glascow Coma Scale, the patient can open his eyes in response to commands speech, (3 out of 4); localizes pain, (5 out of 6); yet produces inappropriate words, (3 out of 5); for a Glascow coma score of 11.

How well did you know this?
1
Not at all
2
3
4
5
Perfectly
38
Q
The most frequent location of internal orbital injury in children seven years old and under is the?
A. floor
B. medial wall
C. lateral wall
D. roof
A

ANSWER: D
RATIONALE:
Fronto-orbital injuries are the most frequent in this age group. Because the antrum, sphenoid, ethmoid and frontal sinuses are not yet pneumatized in this age group, fronto-basilar (or roof) injuries most commonly occur. Wall or floor injuries occur in young children.

How well did you know this?
1
Not at all
2
3
4
5
Perfectly
39
Q
After nasal injuries in children, growth disturbance are most associated with premature ossification of which suture?
A. Nasofrontal
B. Septovomerine
C. Nasomaxillary
D. Nasoethmoidal
A

ANSWER: B
RATIONALE:
The septovomerine suture determines growth in this area. If this suture is involved in trauma growth consequences are a concern since neither of the other sutures, if involved, provides as deforming a growth consequence.

How well did you know this?
1
Not at all
2
3
4
5
Perfectly
40
Q
When reconstructing an orbital floor injury, the safe distance for dissection from the infraorbital rim to the anulus of Zinn is up to how many millimeters?
A. 20
B. 25
C. 35
D. 40
A

ANSWER: D
RATIONALE:
According to a recent cadaveric study involving specimens with intact soft tissue, the mean distance from the inferior orbital rims to the annulus of Zinn is 39.4 mm +/- 2/9 mm. Previously cited studies on dry skulls, using bony landmarks only with no soft tissue references at the orbital apex provide clinically less useful information.

How well did you know this?
1
Not at all
2
3
4
5
Perfectly
41
Q
The most common chronic problem associated with the surgical treatment of frontal sinus fractures is?
A. The development of mucoceles
B. Chronic pain
C. Osteomyelitis
D. Contour deficits and irregularities
A

ANSWER: B
RATIONALE:
With the advent of rigid internal fixation, contour deficits and irregularities are infrequent. Using modern aseptic protocols, osteomyelitis is very uncommon. While mucoceles do occur in rare instances when sinus membrane regenerates, pain remains the most frequent of chronic problems.

How well did you know this?
1
Not at all
2
3
4
5
Perfectly
42
Q
Patients who survive facial fractures are most likely to have suffered what forms of facial injury?
A. Upper only
B. Mid only
C. Lower only
D. Combinations of lower, mid and upper
A

ANSWER: C
RATIONALE:
Mandibular injury is associated with c-spine injury, but mid- and upper are associated with death. Mid- and upper-facial third injuries initially act as shock absorbers until a particular magnitude of impact, after which they transmit force to the neurocranium. The more commonly results in fatality than does trauma to the lower 1/3 of the facial skeleton.

How well did you know this?
1
Not at all
2
3
4
5
Perfectly
43
Q
The intercanthal distance in the uninjured adult patient is approximately:
A. 25-30 mm
B. 31-35 mm
C. 36-40 mm
D. 41-45 mm
A

ANSWER: B
RATIONALE:
The inter-canthal distance in adult Caucasians is 33 + mm. This varies minimally with gender and race. The other answers are outside of this range.

How well did you know this?
1
Not at all
2
3
4
5
Perfectly
44
Q

Which of the following is true of the annulus of Zinn?
A. It is attached to the lacrimal, ethmoid and sphenoid bones.
B. It contains the ophthalmic artery and its branches.
C. It is the origin of the rectus and oblique muscles.
D. It contains the maxillary and ophthalmic divisions of the trigeminal nerve.

A

ANSWER: B
RATIONALE:
The annulus of Zinn contains the ophthalmic artery and its branches. The oblique muscles originate outside of the annulus. The annulus is distant from the lacrimal bone, and does not contain the maxillary division of the trigeminal nerve.

How well did you know this?
1
Not at all
2
3
4
5
Perfectly
45
Q

Which of the following is the best definition in the Markowitz and Manson classification system of a Type II nasoethmoidal injury?
A. a single large fragment, with the canthal ligament attached. B. minor comminution, with the canthal ligament attached.
C. comminution beneath the canthal ligament.
D. comminution with the canthal ligament detached.

A

ANSWER: B
RATIONALE:
Below are figures adapted from the original article on nasoethoidmal fractures published by Markowitz and Manson in 1991. Choice B is the correct answer. Choice A represents a Type I fracture. Choice C doesn’t fit into any classification scheme since it doesn’t address the region of the canthal ligaments. Choice D represents a Type III fracture. This figure depicts the nasoethmoidal region. The region shaded is the central fragment. The inset figure represents a Type I fracture, where the canthal ligament is attached to a large central fragment and there is no comminution of the fractured nasoethmoidal region. This figure represents a Type II fracture. There is some comminution of the nasoethmoidal region but the canthal ligament is attached to a substantial fragment of bone. This figure represents a Type III fracture. The canthal ligament is detached or there is severe comminution with the canthal ligament attached to a small fragment of bone.

How well did you know this?
1
Not at all
2
3
4
5
Perfectly
46
Q

When repairing cranial bone trauma utilizing a coronal approach, the temporal branches of the facial nerve can best be preserved by:
A. avoid extending the incision into the preauricular areas
B. confining the surgical dissection between the superficial temporal fascia and the deep
temporal fascia
C. insuring the surgical dissection is deep to the superficial layer of the deep temporal fascia
D. avoid reflecting the periosteum of the zygomatic arch.

A

ANSWER: C
RATIONALE:
The temporal branches lie immediately beneath the superficial layer of the temporal fascia, just above the superficial layer of the deep temoral (temporalis) fascial. Dissection below the superficial temporal fascia this layer may injure the nerve. Dissection beneath the superficial layer of the deep temporal fascia preserves the temporal facial nerve branches.

How well did you know this?
1
Not at all
2
3
4
5
Perfectly
47
Q

Which of the following surgical approaches for repair of orbital trauma has the highest incidence of post operative scleral show?
A. Transconjunctival without lateral cantholysis B. Infraorbital (orbital rim)
C. Upper eyelid blepharoplasty
D. Subciliary

A

ANSWER: D
RATIONALE:
Several studies have shown that the subciliary approach has the highest rate of post operative scleral show. The scar contracture associated with the infraorbital approach does no affect the orbicularis to the same degree as for other incisions. The transconjunctival incision is associated with minimal scleral show. The design of the upper blepharoplasty incision is associated with the least percentage of this complication.

How well did you know this?
1
Not at all
2
3
4
5
Perfectly
48
Q

The most sensitive clinical laboratory indicator able to confirm cerebrospinal fluid leakage is:
A. Comparison of suspected fluid glucose to patient’s serum glucose
B. Dipstick test utilizing glucose oxidase
C. Beta 2 transferrin level of suspected fluid
D. Comparison of protein and potassium levels of suspected fluid to nasal secretions and
serum levels

A

ANSWER: C
RATIONALE:
Beta 2 transferrin is found only in the brain and eyes. The patient’s CSF may be contaminated with blood and therefore mimic serum. The dipstick test is colorimetric and if contaminated with red blood, may alter the results. Again, CSF may be contaminated with nasal secretions or blood and thus mimic those fluids rather than CSF.

How well did you know this?
1
Not at all
2
3
4
5
Perfectly
49
Q
The end metabolic degradation byproducts of bioresorbable osseous fixaton devices (plates and screws) are:
A. glycolic acid
B. lactic acid
C. carbon dioxide and water
D. Acetic acid
A

ANSWER: C
RATIONALE:
Both homopolymer and copolymer products follow the same metabolic degradation pathway culminating in the citric acid cycle, ultimately ending with the production of carbon dioxide and water.

How well did you know this?
1
Not at all
2
3
4
5
Perfectly
50
Q

An adult patient, with a normal dentition, has a closed right mandibular subcondylar fracture with 90 degree medial displacement of the condylar head. Secondary to this fracture, the patient could be expected to demonstrate which of the following clinical findings?
A. Deviation of the mandible to the left with opening
B. Reduced right lateral excursion
C. Right posterior apertognathia
D. Reduced left lateral excursion

A

ANSWER: D
RATIONALE:
With a right subcondylar fracture, the action of the right lateral pterygoid muscle would be reduced and the patient would deviate to the right upon opening, and would experience diminished left lateral excursion. Lastly, decreased right ramus height would cause an occlusal prematurity on the right side.

51
Q

In the case of an isolated lesion of the right oculomotor nerve:
A. The right globe rotates upward and outward.
A. The left eye consensual light reflex is preserved.
B. Motor nerves alone are affected resulting in ptosis and miosis.
C. Light stimulation of the left eye results in a consensual reflex in the right eye.

A

ANSWER: B
RATIONALE:
Motor and sensory nerves are affected. Light stimulation in the left eye produces no consensual reflex in the right eye, since the oculomotor nerve carries parasympathetic branches that allow consensual pupillary constriction. The left eye consensual light reflex is preserved. Only abduction (via cranial nerve VI) and adduction (via cranial nerve IV) of the right globe are possible.

52
Q

Low velocity missile wounds are characterized by:
A. small entry wounds and larger exit wounds.
B. bullet speed over 2000 ft per second
C. minimal tissue avulsion
D. soft tissue cavitation injury

A

ANSWER: C
RATIONALE:
Low velocity wounds are characterized by a small and clean cut or ragged entrance wound but no exit wound. When hard tissue is struck fracture, comminution and displacement occurs, but external avulsion is rare. The soft tissue cavitation is minimal. Low velocity missiles travel at less than 2000 ft/sec.

53
Q

Which of the following best describes the most appropriate initial fluid bolus during the fluid resuscitation of a pediatric patient.
A. 1 liter lactated Ringer’s solution
B. 500 cc normal saline
C. 20 cc/kg body weight crystalloid solution
D. 10 – 15 cc/kg body weight colloid solution

A

ANSWER: C
RATIONALE:
Initial resuscitation for pediatric patients is done with crystalloid solution, and the volume administered is determined by body weight. Fluid resuscitation is not accomplished with set amounts of fluid., nor is colloid an appropriate medium for initial intravascular resuscitation.

54
Q
Along which wall of the orbit is there a normal bony prominence just behind the equator of the globe?
A. Superior and lateral
B. Lateral and inferior
C. Inferior and superior
D. Inferior and medial
A

ANSWER: D
RATIONALE:
The bone anatomy of the orbital walls has been well defined. Often forgotten in
clinical practice, however is the elevation of the infero-medial orbital wall behind the equator
of the globe. In addition, significant alteration of the bony anatomy along the medial wall is sometimes not appreciated because displacement can result in a straight medial wall that to the uninitiated appears normal.

55
Q

In the setting of traumatic blindness, which of the following is an acceptable indication for surgical decompression of the optic nerve?
A. failure to respond to high-dose steroid therapy
B. visual loss immediately following the trauma event
C. penetrating trauma
D. delayed visual loss following blunt trauma

A

ANSWER: D
RATIONALE:
Following blunt trauma visual recovery after decompression is not likely whenever there was immediate blindness, penetrating ocular injury, or failure to respond to the initiation of high-dose steroids.

56
Q
Failure to re-suspend the periosteum overlying the zygoma after fracture repair most commonly results in which of the following deformities?
A. ectropion of the lower eyelid
B. sagging of the facial soft-tissues
C. thinning of the upper lip
D. inward rotation of the vermillion
A

ANSWER: B
RATIONALE:
Ectropion results from overzealous dissection, tissue injury or pexing of the periorbita. Thinning of the lip and inward rotation of the vermillion should only be a consideration if the periosteum was dissected from the anterior maxilla. Following coronal or hemicoronal flap elevation and exposure of the zygomatic bone and malar eminence, posterio-superior resuspension of the periosteum is indicated to prevent tissue sagging.

57
Q
Which of the following is the appropriate amount of time for the arch bar fixation of a dentoalveolar fracture:
A. 3-5 days
B. 7-10 days
C. 14-17 days
D. 21-25 days
A

ANSWER: D
RATIONALE:
Isolated tooth avulsion is treated by splinting for seven tot en days with isolation from occlusal function and endodontic therapy on theeth with fully developed apices. In the case of a true alveolar fracture, three to four weeks, or twenty one to twenty eight days is an appropriate duration of arch bar fixation.

58
Q

When a primary tooth is traumatically intruded one should:
A. extract the tooth.
B. observe for 12 months and extract if it should not re-erupt.
C. splint the tooth 2-3 weeks.
D. observe for 4-8 weeks and extract if it should not re-erupt.

A

ANSWER: D
RATIONALE:
Immediate extraction does not give the tooth any chance for survival. If splinted in the intruded position, the tooth is condemned to a malposition. If splinted in the proper position, the expanded alveolus would not permit intimate root contact between the surrounding alveolus and periodontal ligament remnants on the root and therefore preclude survival. Observation for a year indicates that the tooth is ankylosed. Observation for 4-8 weeks and then extraction if no re-eruption is observed is the most appropriate answer.

59
Q
The initial antibiotic coverage for a cat, dog, or human bite is:
A. penicillin
B. clindamycin
C. cephalexin
D. amoxicillin with clavulanate
A

ANSWER: D
RATIONALE:
Augmentin. This antibiotic is bacteriocidal for the range and spectrum of human and animal bite pathogens including Staphylococcus species and Pasteurella multocida

60
Q

Traumatic disruption of the adult nasolacrimal system is best handled acutely by:
A. Silicone nasolacrimal duct intubation x 3-4 months
B. Cannulation with Bowman probe of the inferior canaliculus
C. Cannulation with Bowman probe of the superior canaliculus
D. Dacryocystorhinostomy

A

ANSWER: A
RATIONALE:
Nasolacrimal duct intubation may bypass a disrupted nasolacrimal apparatus and avoid the morbidity associated with a dacryocystorhinostomy. Dacryocystorhinostomy is reserved for a chronic condition. Cannulation should be instituted for both inferior and superior canaliculi.

61
Q
43. A 22 year old male status post two gun shot wounds to the face and abdomen presents with a blood pressure of 80/40 and a pulse of 160 after 2 liters of Ringers lactate. There is no significant facial hemorrhage or expanding hematoma. Proper immediate management is:
A. four vessel angiography of the neck
B. CT scan of the abdomen
C. diagnostic peritoneal lavage
D. emergency celiotomy
A

ANSWER: D
RATIONALE:
This patient exhibits no symptoms of massive hemorrhage from the facial gunshot wounds, and is hemodynamically unstable with a penetrating abdominal injury. This is a clssic indication for an emergency celiotomy for intra-abdominal hemorrhage control. CT abdominal scans or diagnostic peritoneal lavage may be indicated in blunt tauma with evidence of intra- abdominal hemerrhage; but these procedures are time-consuming and are indicated if there is greater hemodynamic control. From the clinical presentation, although eventual head and neck angiography may be indicated, the more emergent problem is shock from abdominal blood loss and must be addressed first by cleotomy.

62
Q

The following demonstrates facial lacerations, which of the following most closely follows the resting skin tension lines?

A

ANSWER: B
RATIONALE:
‘The concept of resting skin tension lines (RSTLs) was introduced by Borges and Alexander in 1962. They based their definition of these RSTLs on how a wound tends to behave following incision of the skin, i.e., whether the wound gapes widely or very little. It has been known for decades (Dupuytren, 1834, Langer 1861) that skin incisions placed right angles to skin tension lines produce widely gaping wound margins. If these lines are followed during the incisions or excision of scars and neoplasms generally a far better result will be achieved than if the incisions cross the tension lines. The course of the facial tension lines does not always coincide with normal wrinkle lines due to aging.’
The answer is B. All of the other lacerations are at right angles to the RSTLs

63
Q
You consult on a patient status post motor vehicle accident and see that a neurosurgery consult has noted the presence of epidural hematoma with transtentorial herniation. Which of the following would not be typical symptoms?
A. Ipsilateral fixed, dilated pupil
B. Decerebration
C. Decorticate posturing
D. Coma
A

ANSWER: C
RATIONALE:
Epidural hematoma, usually caused by a middle meningeal arterial bleed, occurs between the dura mater and the inner table of the calverium. The classic triad of trastentorial herniation includes decerebrate posturing (extension of the arm at the elbows with internal arm rotation), a fixed and dilated pupil on the side of the herniation, and coma. Decerebrate posturing indicates neurologic damage at or below the midbrain. Decorticate posturing indicates severe neurologic damage in the hemisphere above the midbrain; and is clinically typified by arm flexion and fist clenching.

64
Q
Which of the following is not a treatment common to traumatic limb crush injury, severe electrocution, and malignant hyperpyrexia?
A. calcium chloride
B. sodium bicarbonate
C. mannitol
D. spironolactone
A

ANSWER: D
RATIONALE:
All three of theses musculoskeletal injuries are typified by rhabdomyolysis and electrolyte disturbances including hyperkalemia and metabolic acidosis. Complications include renal failure from myoglobin precipitation in nephrons, metabolic derangements from acidosis, and hyperkalemia. Treatment includes minimization of myoglobin-induced renal impairment by fluid expansion and osmotic diuresis by mannitol to minimize myoglobin deposition, sodium bicarbonate to treat metabolic acidosis and to alkalinize the urine to promote myoglobin solubility/excretion. Cardiac sequelae of hyperkalemia may be emergently opposed by intravenous calcium administration. Spironolactone, a potassium-sparing diuretic, would not be included in such treatment, although a loop diuretic such as furosemide may be useful.

65
Q

When repairing eyelid lacerations or avulsive injuries:
A. The eyelid skin should be aggressively débrided.
B. The lid margin is initially approximated with a suture at the “gray line”.
C. Repair must be completed within 24 hours.
D. Split thickness skin grafts are best for replacing avulsed eyelid skin.

A

ANSWER: B
RATIONALE:
The best answer is to re-approximate landmarks by suturing the lid at the gray line. A lid may be repaired by a number of techniques, each of which has a specific time limitation, therefore 24 hours may vary. Aggressive debridement should be avoided. Split thickness skin grafts are not the most desirable modality as they may have texture and color differences.

66
Q
Which of the following is not seen in cases of fracture of the larynx with acute airway obstruction.
A. Hoarseness
B. Palpable fracture
C. Dysphasia
D. Dyspnea
A

ANSWER: C
RATIONALE:
Dysphagia, pain on swallowing, is not part of the classic triad described by the other answers: a palpable fracture, . hoarseness (if the vocal chords are compromised); and comprised breathing.

67
Q
Which of the following is a common finding in patients with bilateral displaced fractures of the condylar processes treated by closed reduction that is not commonly seen following proper open reduction?
A. Temporomandibular joint ankylosis
B. Loss of posterior facial height
C. Temporomandibular disc displacement
D. Facial muscle weakness
A

ANSWER: B
RATIONALE:
Disc displacement may occur from the injury or the treatment, but has not been shown to be significantly different for the two different types of treatment. Likewise, the incidence of ankylosis has not been shown to be statistically different with either treatment. Facial weakness via cranial nerve V damage is only possible in the setting of open reduction.

68
Q
A mandibular angle fracture with comminution, infection, or loss of bone buttressing is best treated with which type of fixation:
A. miniplate
B. dynamic compression plate (DCP)
C. reconstruction plate
D. wire osteosynthesis
A

ANSWER: C
RATIONALE:
The need for absolute stability for these types of fractures negates wire or miniplate fixation. DCP causes interfragmentary compression with possible bone devitalization and necrosis. The reconstruction plate is specifically indicated in these instances.

69
Q
  1. Which of the following would be observed in a patient with an isolated C4-5 spinal cord injury?

A. Disturbance of heart rate
B. Apnea
C. A major loss of diaphragmatic function
D. An air embolism

A

Answer: C
Rationale:
The phrenic nerve innervates the diaphragm and arises from cervical segments 3, 4, and 5. Therefore, a cervical spine injury at this level would likely cause severe ventilatory dysfunction, affecting tidal volume, vital capacity, or FEV1.
A is incorrect, since parasympathetic innervation to the heart arises from Cranial Nerve X and the sympathetic innervation arises from the cervical sympathetic trunk, and both join to form the cardiac plexus.
B is incorrect, since the injury only involves C4-5 (and not C3); the phrenic nerve would probably still be somewhat functional, and some ventilation would be expected.
D is incorrect. This is a completely random answer, not necessarily related to this injury.

Reference:
Hollingshead’s Manual of Practical Anatomy

70
Q

A 16-year-old male is involved in a motor vehicle accident. He is found unresponsive at the scene and is intubated and brought to the emergency room. On arrival he both opens his eyes and withdraws only to painful stimulus. What is his Glascow Coma Scale classification?

A. 4T
B. 5T
C. 6T
D. 7T

A
Answer: C
Rationale:
Intubated patients receive a 'T' since they are unable to verbally respond
Best Eye Response. (4)
No eye opening.
Eye opening to pain.
Eye opening to verbal command. Eyes open spontaneously.
Best Verbal Response. (5)
1. No verbal response
2. Incomprehensible sounds.
3. Inappropriate words.
4. Confused
5. Orientated
Best Motor Response. (6)
1. No motor response.
2. Extension to pain.
3. Flexion to pain.
4. Withdrawal from pain.
5. Localising pain.
6. Obeys Commands.
Reference:
ATLS Manual, 1997
71
Q

When utilizing the Champy technique for fixation of a mandibular angle fracture, the plate used for fixation is:
A. an eccentric dynamic compression plate.
B. placed in the zone of compression.
C. placed in the zone of neutrality.
D. placed in the zone of tension.

A

Answer: D

Rationale:
The principle of the Champy technique is for fixation in the zone of tension with natural functional forces aiding in the approximation of the fracture in the area of compression.

Reference:
Champy M, Lodde JP, Scmitt R, et al.

72
Q

When utilizing a retromandibular approach to a subcondylar fracture,which of the following is true?
A. The dissection can go behind or through the parotid gland
B. The dissection goes between the temporal and zygomatic branches of the facial nerve
C. The approach is best for subcondylar fractures that are high
D. The superior extent of the incision begins 2 cm below the earlobe

A

Answer: A
Rationale:
Dissections posterior to the parotid and through the parotid have been described. Classically the dissection carried between the cervicofacial trunk and the temporofacial trunk of the facial nerve. The dissection may be on either side of the marginal mandibular branch. This approach is best for lower subcondylar fractures and more limited for higher fractures. The incision begins approximately 0.5 cm below the earlobe.

Reference:
Hinds EC: Correction of prognathism by subcondylar osteotomy. J Oral Maxillofac Surg 16:209, 1958
Ellis E III, Zide MF: Surgical Approaches to the Facial Skeleton. Baltimore. Lippincott Williams & Wilkins. 1995

73
Q

When performing a Risdon approach to a mandibular angle fracture:
A. posterior to the facial artery the marginal mandibular branch of the facial nerve is always located below the inferior border of the mandible.
B. posterior to the facial artery the marginal mandibular branch of the facial nerve may drop 2.5cm below the inferior border of the mandible.
C. anterior to the facial artery the marginal mandibular branch of the facial nerve is found below the inferior border of the mandible less than 10% of the time.
D. anterior to the facial artery the marginal mandibular branch of the facial nerve usually has only one branch.

A

Answer: C

Rationale:
The two classic articles on this subject quote incidences of 0% and 6% frequency of the marginal mandibular branch of the facial nerve being found below the inferior border of the mandible anterior to the facial artery. Posterior to the facial artery, 19% of the time the marginal mandibular branch of the facial nerve may be located below the inferior border of the mandible up to 1 (or1.2) cm. Anterior to the facial artery the marginal mandibular branch of the facial nerve has one branch only 21% of the time.

Reference:
Ziarah HA. Atkinson ME. The surgical anatomy of the mandibular distribution of the facial nerve. [Journal Article] British Journal of Oral Surgery. 19(3):159-70, 1981 Sep
Dingman R, Grabb W: Surgical anatomy of the mandibular ramus of the facial nerve based on the dissection of 100 facial halves. Plast Reconstr Surg 29:266-272, 196

74
Q

When treating uncomplicated, compound mandible fractures, the current recommendation for antibiotics is that:
A. antibiotic coverage is not needed.
B. perioperative antibiotic use is indicated.
C. pre-op antibiotics with a 7 day post-operative course are indicated.
D. antibiotics are only useful for open reductions.

A

Answer: B
Rationale:
Compound or open mandible fractures have been shown to have reduced infection rates when perioperative antibiotics have been used. The use of postoperative courses of antibiotics has not been shown to affect the rate of infection.
Reference:
Zallen RD, Curry JT:A study of antibiotic usage in compound mandibular fractures. J Oral Surg; 33:431, 1975.
Abubaker AO, Rollert MK:Postoperative antibiotic prophylaxis in mandibular fractures:A preliminary randomized double-blinded and placebo controlled clinical study. J Oral Maxillofac Surg; 59:1415, 2001.

75
Q

Surgical exposure of which of the following areas would require soft tissue re-suspension to re- establish facial form?
A. Midface exposure via an oral vestibular incision
B. Symphysis exposure via submental incision
C. Zygomatic exposure via coronal incision
D. Orbital floor exposure via transconjunctival incision.

A

Answer: C

Rationale:
Extended exposure of the facial skeleton is indicated for management of many complex facial fractures. Re-suspension of the soft tissue is important to establish proper facial form. Midface exposure via vestibular incision avoids key suspensory areas, thus simple closure is usually indicated. Symphysis exposure via a submental incision leaves the origin of the mentalis muscle attached to the surround soft tissue and/or bone. Re-suspension via this approach is not indicated. With exposure of the orbital floor via a transconjunctival incision simple closure is all that is indicated. When the zygomatic process is exposed via a coronal approach the superficial layer of the temporalis fascia is incised and periosteum overlying the arch is detached. This can result in ptosis of the suborbicularis oculi fat pad. This fat pad must be re-suspended to the lateral orbit.

Reference:
Manson PN, Clark N, Roberston B, et al Subunet Principles in Midface Fractures:The Importance of Sagittal Buttresses, Soft Tissue Reductions and Sequencing Treatment of Segmental Fractures. Plast Reconstr Surg 103:1287-1306, 1999.

76
Q

Which of the following is the treatment of choice in management of bilateral mandibular body fractures in a patient with an extremely atrophic mandible (less than 6 mm of bone height)?
A. Closed reduction with a gunning splint
B. Lag screw fixation of the mandibular segments
C. Fixation with mini plates
D. Fixation with reconstruction plates

A

Answer: D

Rationale:
In order to allow healing of a fracture of the atrophic mandible, the fixation technique must neutralize the tension forces on the mandible. The only fixation technique that satisfies these criteria is the use of a reconstruction plate.

Reference:
Sikes JW, Smith BR, Mukherjee DP. An In-Vitro Study of the Effect of Bony Buttressing on Fixation Strength of a Fractured Atrophic Edentulous Mandible Model J Oral Maxillofacial Surgery 58:56-61, 2000.

77
Q
  1. A patient presents with a displaced inner and outer table frontal sinus fracture with associated CSF leak. There is no involvement of the nasofrontal drainage system. Which of the following procedures is indicated for the management of the frontal sinus?
    A. Open reduction and internal fixation of the inner and outer table of the frontal sinus
    B. Obliteration of the frontal sinus with preservation of the nasofrontal drainage system
    C. Cranialization of the frontal sinus with obliteration of the nasofrontal draining system
    D. Obliteration of the frontal sinus with obliteration of the nasofrontal drainage system
A

Answer: C

Rationale:
Fracture of the inner and outer table of the frontal sinus with communition and CSF leak represent the most complex of frontal sinus injuries. The dual tears associated with the frontal sinus leak must be repaired and they are usually approached via a coronal incision with the removal of a portion of the frontal bone. With communition of the posterior table ORIF of these fractures would be difficult. Because of the rapid growth of respiratory mucosa any and all remnants of epithelium within the sinus must be removed and the bony walls curetted to remove any epithelium remnants. Obliteration of the sinus is indicated when there is an intact posterior sinus wall. This allows for maintenance of the material used for obliteration to remain within the sinus cavity. In all cases of obliteration of the frontal sinus, obliteration of the nasofrontal ducts is also indicated to prevent respiratory microflora from entering into this cavity or entering into the sinus, thus preventing infection. In this case, due to the fracture of the inner and outer table of the frontal sinus with associated CSF leak, all of the posterior table should be removed to allow for expansion of the frontal lobes into the residual cavity. The outer table is reconstructed in this procedure. In all cases of cranialization of the frontal sinus, the nasofrontal drainage system is obliterated.

Reference:
Rohrich RJ, Hollier LH. Management of Frontal Sinus
Fractures; Changing Concepts. Clinics in Plastic Surgery 19:219-232:1

78
Q
Penetrating trauma located just above the clavicles would be an injury to what zone of the neck?
A. Zone 1
B. Zone 2
C. Zone 3
D. Zone 4
A

Answer: A

Rationale:
The zones of the neck were originally described by Dr. Monson in
1969. Zone 1 is defined as the area from the clavicles to the cricoid cartilage. The risk of injury to the great vessels is common in this area. Zone 2 represents the area from the cricoid cartilage to the angle of the mandible. It is the largest area of the neck and thus is most likely to be injured with penetrating neck trauma. Zone 3 represents the region from the angle of the mandible to the base of the skull. The area presents the most difficult area for surgical access.

Reference:
Reference:Monson DO, Saletta JD, Freeark RJ. Carotid Vertebral Trauma. J Trauma 9:987- 99, 1969.

79
Q
At what distance behind the superior aspect of the medial orbital rim would the anterior ethmoidal foramen and its associated artery be located?
A. 0to5mm
B. 10 to 15 mm
C. 20 to 25 mm
D. 30 to 35 mm
A

Answer: C

Rationale:
The anterior and posterior ethmoidal arteries will require identification when performing medial orbital wall or roof dissections. The vessels may also be identified for ligation in order to control nasal bleeding. The anterior ethmoidal foramen transmits the anterior ethmoidal artery and anterior ethmoidal branches from the nasociliary nerve. The posterior ethmoidal foramen transmits the posterior ethmoidal artery and the spheno-ethmoidal nerve from the nasociliary nerve. These foramina can be located posterior to the junction of the medial orbital wall and orbital roof. The anterior ethmoidal foramen is located twenty to twenty-five millimeters behind the medial orbital rim and the posterior ethmoidal foramen is located twelve millimeters posterior to this point or approximately thirty-two to thirty-seven millimeters.

Reference:
Ochs MW. Orbital and Ocular Trauma. Miloro M (Ed):Peterson’s Principals of Oral and Maxillofacial Surgery, Second Edition. BC Decker, Inc. Hamilton, Ontario 2004. p. 465

80
Q
A 9-year-old child is brought to the emergency room after being struck by a car. The patient is unresponsive to command and breathing only infrequently. Cervical immobilization is in place. Oral intubation attempts are unsuccessful because of brisk bleeding from facial fractures. The most appropriate next step is:
A. obtain an angiogram
B. surgical cricothyroidotomy.
C. surgical tracheostomy.
D. percutaneous tracheostomy.
A

Answer: B & C

Rationale:
The child is too young for cricothyroidotomy and percutaneous tracheostomy is not indicated for emergency airway. Cricothyroidotomy is not recommended in children less than 12 years of age, since the cricoid cartilage is the only circumferential support to the upper trachea. ATLS teaches that a child may be temporarily oxygenated with needle jet insufflation, but this technique does not provide ventilation and is not a definitive airway. Although surgical tracheostomy is unappealing in this age category, it is the procedure of last resort in this scenario.

Reference:
Advanced Trauma Life Support for Doctors Course Student Manual, 7th Ed., First Impression Press, 2004, page 48,49,250

81
Q

An adult patient has a self-inflicted shot gun wound to the mouth. He is brought to the emergency room intubated with a large facial wound. Vital signs upon arrival are heart rate 130, respiration 35, and blood pressure 90/60. The patient is fighting restraints. A urinary catheter insertion produced only 10 cc of urine. Which of the following Class of hemorrhage and initial resuscitation treatment is correct?
A. Class II. Treat with crystalloid solution
B. Class III. Treat with blood transfusion
C. Class III. Treat with crystalloid solution and blood transfusion
D. Class IV. Treat with crystalloid and blood transfusion.

A

Answer: C

Rationale:
Class III hemorrhagic shock is characterized by tachycardia, tachypnea, mental status changes and measurable fall in systolic blood pressure, but without the significant narrowing of pulse pressure seen in Class IV shock. Also the tachycardia and tachypnea are worse in Class IV shock. Lactated Ringers or 0.9% normal saline is the fluid of choice for initial resuscitation. Given excessive hemorrhage, >20%, red blood cells should be replaced to maintain oxygen carrying capacity. Cross-matched is the ideal choice, but time constraints (30 min) may require Type-specific blood (5-15 min) to be used.

Reference:
Advanced Trauma Life Support for Doctors Course Student Manual, 7th Ed., First Impression Press, 2004, page 75
Miloro M, et al. Peterson’s Principles of Oral and Maxillofacial Surgery, 2nd Edition. Pg 343. BC Decker Inc, 204.

82
Q

When opening a subcondylar fracture through a retromandibular approach:
A. the patient should be paralyzed .
B. the forehead should always be in view in the surgical field.
C. the closure of the parotid capsule/SMAS and platysma layer is critical to minimize the
occurrence of a salivary fistula.
D. the local anesthesia should be injected deep to the platysma muscle.

A

Answer: C

Rationale:
Salivary-cutaneous fistulae are potential sequelae to the retromandibular approach. To help avoid this complication it is recommended that a “water tight” closure be performed.

Reference:
Ellis E. Zide M. Surgical Approaches to the Facial Skeleton Williams and Wilkins, Media, PA, 1995

83
Q
Intravenous fluids administered to a patient with acute brain injury should be:
A. hypotonic.
B. hypertonic.
C. normotonic.
D. glucose-containing.
A

Answer: C

Rationale:
Intravenous fluids administered to acutely brain-injured patients should normotonic in order to maintain normovolemia. Hypovolemia and hypervolemia is harmful to these patients. Hyperglycemia from use of glucose-containing fluids is harmful to the injured brain. Normal saline or Ringer’s lactate is recommended for resuscitation. Serum sodium levels need to be monitored to prevent hyponatremia which can lead to brain edema.

Reference:
Advanced Trauma Life Support for Doctors. Student Course Manual, 7th Edition, p.165. American College of Surgeons 2004.

84
Q
Regarding blunt force trauma to the mid face, which of the following anatomic structures can be considered a pillar of support that can resist compression and deformation?
A. Anterior maxillary sinus wall
B. Posterior maxillary sinus wall
C. Zygomatico-maxillary junction
D. Zygomatico-sphenoid junction
A

Answer: C

Rationale:
There are three vertical buttresses of the midface:zygomatico-maxillary, pterygo-maxillary, and naso-maxillary. These structures consist of thicker supporting regions of bone that transmit and/or resist forces to the base of the skull. The horizontal buttresses of the midface are the (1) pyriform aperture, (2) maxillary alveolus and palate, (3) orbital rims and (4) base of the skull.

Reference:
Fonseca, RJ, Oral and Maxillofacial Trauma Vol 1, Advances in Maxillofacial Trauma Surgery, Elsevier 2005.
Manson P, et al. Plastic and Reconstructive Surgery. Structural Pillars of the Facial Skeleton:An Approach to the Management of Le Fort Fractures. July 1980, 66(1) pg. 57.

85
Q
Stabilization of the medial canthal tendon during a medial canthopexy should be directed at which of the following directions?
A. Anterior and superior
B. Anterior and inferior
C. Posterior and superior
D. Posterior and inferior
A

Answer: C

Rationale:
Repair of detached medial canthal tendons during a severe naso-orbital ethmoidal fracture repair involves reattachment of one or both tendons to the posterior lacrimal crest in a slightly posterior and superior direction to overcome the forces of migration, relapse, and telecanthus.

Reference:
Fonseca, RJ, Oral and Maxillofacial Trauma Vol 2, Diagnosis and Treatment of Midface Fractures, Elsevier 2005

86
Q
An injury to the temporo-facial trunk of the facial nerve is suspected within a deep cheek laceration. Which of the following will best confirm an intact and functioning facial nerve?
A. A normal Bell’s phenomenon
B. An intact corneal reflex
C. Absence of divergent strabismus
D. Absence of an oculo-cardiac reflex
A

Answer: B

Rationale:
Integrity of the temporo-facial trunk of the facial nerve can be assessed by performing a corneal reflex (tearing to touching the ipsilateral cornea.) The afferent and efferent limbs of the reflex involve the first division of the trigeminal nerve and facial nerve respectively. Strabismus is associated with defects in cranial nerves III, IV, and VI.

Reference:
Bates, B, A Guide to Physical Examination and History Taking, The Nervous System, JB Lippincott 2002

87
Q

Which of the following would be an indication for frontal sinus obliteration?
A. Non-displaced greenstick fracture of the anterior table.
B. Fracture of the posterior table.
C. Nasofrontal duct occlusion.
D. Nasolacrimal duct damage.

A

Answer: C

Rationale:
Displaced anterior table fractures should be opened with careful inspection of the sinus mucosa. A non-displaced posterior table fracture without CSF leak can be managed without obliteration of the frontal sinus. Occlusion of the nasofrontal ducts requires removal of all sinus mucosa from the frontal sinus, inversion of the mucosa into the ducts, and elimination of dead space. This eliminates the possibility of developing a hematoma or seroma. Fat has been historically used with most success; no filler, hydroxyapatite, glass wool, bone, cartilage, muscle, absorbable gelatin sponge and fabric, acrylic have also been used.

Reference:
Miloro M, et al. Peterson’s Principles of Oral and Maxillofacial Surgery, 2nd Edition. Pg 496-501. BC Decker Inc, 204.

88
Q
Which of the following surgical approaches to the inferior orbital rim has the least potential for producing post-operative ectropion and poor cosmesis?
A. Infraorbital
B. Subciliary
C. Transconjunctival
D. Inferior Lid
A

Answer: C

Rationale:
Infraorbital or rim incision results in the worst esthetics and offers no advantages. The subciliary incision has an ectropion rate of approximately 6 %, whereas the transconjunctival incision has an ectropion/entropion rate of approximately 1.2 %. Thus, the transconjunctival incision as the approach of choice to avoid ectropion.

Reference:
Miloro M, et al. Peterson’s Principles of Oral and Maxillofacial Surgery, 2nd Edition. BC Decker Inc, 2004.

89
Q
Which of the following is the most common early complication related to frontal sinus trauma?
A. Mucocele formation
B. Brain abscess
C. Mucopyocele formation
D. Meningitis
A

Answer: D
Rationale:
All of the above cited answers are potential complications to frontal sinus fractures. Meningitis is the most common EARLY complication. The remainders of the answers are delayed or late potential complications. Frontal sinus complications can occur many years after injury (often greater than 20 years after injury). The most serious complications: can include displacement of the frontal bone in the brain and floor of frontal sinus into the orbit. Infectious complications, such as meningitis or intracranial abscess occur via bacterial spread through posterior table fractures or the diploe veins of Behçet; or by osteomyelitis, or mucocoele formation. Mucoceles are the most common chronic complication with diagnosis made via CT scanning. Periodic review by CT should be done every 1, 2 and 5 years following surgery. Symptoms include headache and can include loss of smell (anosmia.).

Reference:
Fonseca, RJ, Oral and Maxillofacial Trauma, Third Edition, 2005, Vol 2, pp 731-32. Miloro
M, et al. Peterson’s Principles of Oral and Maxillofacial Surgery, 2nd Edition. Pg 503. BC Decker Inc, 204.

90
Q

A comminuted angle fracture of the mandible would be treated most effectively with the following fixation:
A. ORIF with reconstruction bone plate applied to the inferior border with 3 bone screws on each side of the fracture.
B. antibiotics and soft diet for 4 weeks.
C. anatomic reduction with Champy bone plate at the superior border.
D. maxillo-mandibular fixation to restore occlusion for 4 weeks.

A

Answer: A
Rationale:
The reconstruction plate is designed to bear the entire load of the mandibular movement across the fracture site. The remaining methods mentioned do not adequately stabilize the fracture segments and increase the possibilities for non-union and infection.

Reference:
Miloro, M. et al, Peterson’s Principles of Oral and Maxillofacial Surgery, 2nd Edition, page 375, BC Decker INC, 2004

91
Q

The best technique to manage a 40% traumatic avulsion of the upper eyelid is:
A. allow the wound to heal by secondary granulation.
B. undermine and close primarily.
C. split thickness skin graft from the ipsilateral postauricular region.
D. full thickness skin graft harvested via blepharoplasty approach from the contralateral
upper eyelid.

A

Answer: D

Rationale:
Full thickness skin graft from the opposite upper lid is preferred due to similar skin thickness, color, texture and availability with minimal donor site morbidity. This avoids scar and ectropion.

Reference:
Sandler NA, Evaluation and management of traumatic eyelid injury, OMS Knowledge Update vol 3, AAOMS, 2001

92
Q

The most predictable means to treat a large posterior auricular skin/cartilage avulsion is:
A. reattachment as a composite graft with hyperbaric oxygen.
B. microvascular reimplantation.
C. full thickness skin graft to cover exposed cartilage.
D. 2 phase bipedicled postauricular flap.

A

Answer: D

Rationale:
Exposed cartilage will not accept a skin graft. Composite free auricular grafts and microvascular reanastomosis have poor success rates due to poor venous drainage. The bipedicled flap nicely and predictably reconstructs the posterior helical defect. If the avulsed segment is 1 cm or less it can be reattached and allowed to revascularize. Larger avulsive injuries, the “pocket principle” can be used where the detached ear is dermabraded and reattached to the stump and then buried under a skin flap in the posterior auricular region to provide vascular supply. 2-3 weeks later the revascularize ear is uncovered and allowed to reepithelialize.

Reference:
Dierks EJ, Deeb GR, Evaluation and management of ear injuries, OMS Knowledge Update vol 3, AAOMS, 2001

93
Q

Recurrent sialocoele formation after emergency room repair of a deep facial laceration is best handled by:
A. multiple aspirations, pressure dressings, and antisialagogues.
B. superficial parotidectomy.
C. reexploration of the wound, cannulation of Stensen’s duct, and repair of the parotid
capsule.
D. low dose radiation.

A

Answer: A & B

Rationale:
Sialoceoles that repeatedly reform, even after aspiration, and pressure dressings, and antisialogogues may indicate a parotid capsular or Stensen’s duct injury that requires early reexploration and repair. Sialocoele formation within 48 hours can indicate an injury to the salivary ductal system and exploration of the injury is needed. The patient also needs to have antibiotic coverage with penicillin or a cephalosporin.

Reference:
Fonseca RJ and Walker RV, “Management of soft tissue injuries”, Oral and Maxillofacial Trauma, vol 1, pg. 642. WB Saunders, 1991

94
Q

A 5cm. full thickness scalp avulsion down to the cranium is best treated by a:
A. split thickness skin graft.
B. full thickness skin graft.
C. split thickness skin graft placed over bur holes made in the outer table.
D. rotation-advancement flap.

A

Answer: D

Rationale:
Exposed cortical cranial bone will not accept skin grafts. Rotational-advancement flaps with multiple galeal releasing incisions parallel to the long axis of the flap will provide adequate coverage in most instances.

Reference:
Fonseca RJ and Walker RV, Management of soft tissue injuries, Oral and Maxillofacial Trauma, vol 1, WB Saunders, 2001

95
Q
In order to avoid irreversible vision loss, an orbital fracture associated with optic nerve compression must be addressed within which length of time?
A. 24 hours
B. 36 hours
C. 4 hours
D. 2 hours
A

Answer: D

Rationale:
The retina is an extension of the brain, and will have similar reaction in the presence of ischemia. Signs and symptoms of visual impairment, severe ocular pain should be evaluated for retrobulbar hematoma. Computed tomography evaluation can demonstrate signs of retrobulbar hematoma formation, but immediate treatment with a lateral canthotomy based on clinical symptoms and signs should be completed prior to a lengthy CT scan. Other signs of optic nerve compromise include (1) afferent papillary defect (2) visual field loss and (3) color vision defects.

Reference:
Girotto JA, Gamble WB, Robertson B, et al, Blindness After Reduction of Facial Fractures, Plast Reconstr Surg 102:1821, 1998.
Miloro M, et al. Peterson’s Principles of Oral and Maxillofacial Surgery, 2nd Edition. Pg 472. BC Decker Inc, 204.

96
Q
After surgical treatment of a zygomaticomaxillary complex fracture,the immediate treatmentof a retrobulbar hematoma accompanied by loss of visual acuity and increased intraocular pressure is:
A. decompression of the frontal sinus.
B. intravenous corticosteroids.
C. lateral canthotomy.
D. medial canthotomy.
A

Answer: C

RATIONALE: The emergency procedure of choice for acute visual acuity loss associated with acute orbital compartment syndrome is dissection of the lateral canthus and disinsertion of at least the inferior crus of the lateral canthal tendon, which allows complete mobility of the lower lid. Visual loss without clear signs consistent with increased IOP is not an indication for this procedure. Other primary indications for lateral canthotomy and cantholysis include an IOP greater than 40 mm Hg and proptosis, which may be used as a criterion for unconscious patients whose visual acuity cannot be determined. Secondary criteria include afferent pupillary defect, ophthalmoplegia, cherry-red macula, optic nerve head pallor, and severe pain, but these are all considered less sensitive or very late signs. Contraindication for this procedure would be a suspected ruptured globe.

REFERENCE: Fonseca, RJ, Walker, R, Oral and Maxillofacial Trauma Volume 1, WB Saunders, 1991

97
Q

To restore proper projection of facial width and orbital volume in a patient with a high energy grossly comminuted zygomaticomaxillary fracture one should:
A. use a Carroll- Girard screw placed transcutaneous to reduce the fracture in a closed method.
B. align the comminuted segments of the infraorbital rim to assure proper placement of the zygoma.
C. expose the sphenozygomatic suture to aid in anatomic reduction.
D. expose and plate the zygomatic-frontal suture area before exposing other areas of the
zygoma.

A

Answer: C

Rationale:
A grossly comminuted zygomatic complex fracture will not be stable with a closed technique. Comminuted segments of the infraorbital rim are a poor guide to placement of the zygoma. The zygomatic-frontal suture is a poor reference for placement of the zygoma. In high energy grossly comminute fractures, the sphenozygomatic suture with its broad contact can aid in the anatomic reduction of the fracture.

Reference:
Baley JS, Goldwasser MS, Management of Zygomatic complex fractures. In: Miloro M ed, Peterson’s Principles of Oral and Maxillofacial Surgery Second edition BC Deker, London 2004 Chapter 23.2 pp 452

98
Q

Irreversible ischemic damage to the retinal tissue secondary to pressure and hypoxia has been shown:

A. within 60 minutes to 2 hours following a traumatic event.
B. avoidable by use of megadose steroids which should be initiated 48 hours after injury.
C. to respond better to treatment when the patient has no initial light perception.
D. to respond better when there is penetrating trauma versus blunt trauma.

A

Answer: A

Rationale:
The optic nerve and retinal tissue, unlike skeletal muscle, have been shown to be extremely sensitive to pressure and hypoxia. Irreversible ischemic damage often occurs within 60 minutes and certainly within 2 hours. Megadose steroids have been recommended during the first 8 hours following injury. The outcome in patients with traumatic optic neuropathy with no light perception on presentation is a poor prognostic factor. Likewise patient presenting with traumatic optic neuropathy after penetrating trauma have a worse prognosis than those with blunt facial trauma.

Reference:
Wang BH, Robertson BC, Girotto JA, Liem A, Miller NR, Iliff N, Manson P, Traumatic optic neuropathy: A review of 61 patients. Plastic Reconstructive Surg 2001; 107:1653-1664

99
Q

Rebleeding following a traumatic hyphema:
A. is associated with the use of topical cycloplegics.
B. can be prevented by increasing intraocular pressure.
C. may occur in 5 to 30% of patients.
D. is increased with the use of carbonic anhydrase inhibitors.

A

Answer: C
Rationale:
The management of a patient with a traumatic hyphema is to use topical cycloplegics and carbonic anhydrase inhibitors to decrease intraocular pressure. Rebleeding can occur in 5- 30% of the patients with hyphema.
Reference:
Baley JS, Goldwasser MS, Management of Zygomatic complex fractures. In:Miloro M ed, Peterson’s Principles of Oral and Maxillofacial Surgery Second edition BC Deker, London 2004 Chapter 23.2 pp 459

100
Q
  1. A CT scan reveals an orbital fracture with a trap door appearance with clinical and radiographic eidence of inferior rectus muscle incarceration is:

A. more likely to occur in adults than in children.
B. warrants early intervention to free up the tissues.
C. can be managed by steroids and delay of surgery for 3 weeks
D. frequently requires the use of an orbital implant to bridge the floor defect.

A

Answer: B

Rationale:
A CT scan of an orbit that has a trapdoor appearance with clinical and radiographic evidence of inferior rectus muscle incarceration is more like to occur in children than in adults and it warrants early intervention to prevent ischemic necrosis to the rectus muscle. Steroids are may be used in cases where there is no evidence of muscle entrapment and mild restricted motility. By its nature, a trapdoor defect seldom requires an implant to bridge a defect.

Reference:
Ochs MW, Orbital and ocular trauma. In: Miloro M ed, Peterson’s Principles of Oral and Maxillofacial Surgery Second edition BC Deker, London 2004 Chapter 24 pp 479

101
Q

51.Calcium phosphate bone cement used for pediatric frontal sinus obliteration:
A. is an almost ideal alloplast for this application.
B. breaks up when placed directly over dura or sinus membranes.
C. is not replaced by bone and remains inert.
D. is readily available and inexpensive.

A

Answer: B

Rationale:
While calcium phosphate bone cement is a near ideal alloplast in that it has unlimited availability, is biocompatible, has high mechanical strength, has a low risk for infection, is radiopaque and is incorporated into the bone, it does have drawbacks. It is expensive, and when placed directly over dura or sinus membranes, it breaks up and a chronic foreign body reaction ensues. This is particularly true for the pediatric population.
Reference:
D’Addario M, Haug RH, Talwar R: Biomaterials for Use in Frontal Sinus Obliteration. Journal of Long-Term Effects of Medical Implants. 14: 455-465, 2004.

102
Q
  1. β2transferrinisfoundonlyin:

A. cerebrospinal fluid, aqueous humor and perilymph.
B. cerebrospinal fluid, serum and aqueous humor.
C. cerebrospinal fluid, nasal secretions and perilymph.
D. serum, nasal secretions and aqueous humor.

A

Answer: A
Rationale:
β2 transferrin is one of two major variants of iron binding transferrin glycoproteins forum in several forms of various body fluids. β2 transferrin lacks a carbohydrate side chain which increases its positive charge, permitting isolation during electrophoresis. β2 transferrin is only found in cerebrospinal fluid, aqueous humor and perilymph. Thus, with the exception of contamination by a ruptured globe, it is an ideal substance to distinguish between cerebrospinal fluid, nasal secretions and serum.

Reference:
Brandt MT, Jenkins WS, Fattahi TT, Haug RH:Cerebrospinal Fluid:Implications in Oral and Maxillofacial Surgery. J Oral Maxillofac Surg. 60:1049-1056, 2002.

103
Q
  1. In the recumbent position, normal intracranial pressure is:

A. 1 to 5 mm Hg.
B. 8 to 12 mm Hg.
C. 15 to 19 mm Hg.
D. 22 to 26 mm Hg.

A

Answer: B

Rationale:
In the recumbent position, intracranial pressure, and thus cerebral spinal fluid pressure, is about 8 to 12 mm Hg or 110 to 150 mm H2O. Autoregulation maintains this pressure.

Reference:
Brandt MT, Jenkins WS, Fattahi TT, Haug RH:Cerebrospinal Fluid:Implications in Oral and Maxillofacial Surgery. J Oral Maxillofac Surg. 60:1049-1056, 2002.

104
Q
55. The cause of permanent visual loss subsequent to traumatic hyphemais:
A. elevated intraocular pressure.
B. retinal artery spasm.
C. traumatic mydriasis.
D. corneal staining.
A

Answer: D

Rationale:
While concomitant injury has been associated with hyphema and permanent visual loss, it is not the primary cause in traumatic hyphema. Retinal artery spasm is not associated with hyphema. Both increased intraocular pressure and traumatic mydriasis are concomitant problems. Corneal staining from myoglobin is the primary cause of permanent loss of vision.

Reference:
Brandt MT, Haug RH:Traumatic Hyphema: A Comprehensive Review. J Oral Maxillofac Surg. 59:1462-1470, 2001.

105
Q

58.The circulatory system for children in trauma differs from adults in that children have a proportionally:

A. higher cardiac output.
B. smaller stroke volume.
C. equal blood volume.
D. decreased physiologic reserve.

A

Answer: B

Rationale:
Children possess a higher cardiac output than adults. Under the physiologic stress of trauma, the child’s stroke volume cannot increase, so that under stress proportionate increases in stroke volume cannot occur and increases in cardiac output are rate determined. Bradycardia could result in hypoxia and hypercapnia, and is ominous. The child’s proportional blood volume is greater than an adult. However, the absolute blood volume and therefore tolerable absolute blood loss is less, so that minor injuries may result in blood loss which is physiologically significant. The child’s blood pressure is maintained through physiologic compensation (vasoconstriction, tachycardia, myocardial contractility). This compensation may be misleading by masking volume reduction. The healthy child has an increased total systemic physiologic reserve, thus with trauma, symptoms of hypoxemia or hypovolemia may not occur until a potentially catastrophic hypovolemia has occurred.

Reference:
American College of Surgeons Committee on Trauma:ATLS Advanced Trauma Life Support
Program for Doctors. 7th Edition, American College of Surgeons, Chicago, IL, 2004, pgs 243-262.

106
Q
59. Cervicalspinefracturesconcomitanttomaxillofacialinjuryaremostoften:
A. “burst’ fractures.
B. “hangman” fractures.
C. Jefferson fractures.
D. subluxations.
A

Answer: D

Rationale:
When considering maxillofacial injury and cervical spine injury, a cause and effect relationship exists between the mandible and cervical spine. There is also a relationship between neurocranial injury and mid- and upper-facial third facial injuries. Approximately one out of every fifty patients with a mandibular fracture will suffer a cervical spine fracture. The vast majority of these are subluxation injuries. When the mandible sustains a force applied to it, the reaction of the body is to move away from the force. This “twisting of the neck, when accentuated, results in a subluxation injury of the cervical spine.

Reference:
Haug RH, Wible RT, Likavec MJ, et al:Cervical spine fractures and maxillofacial trauma. J Oral Maxillofac Surg 49:725-729, 1991.

107
Q
  1. The difference between infected human and animal bites is the presence of:

A. pasteurella multocida.
B. staphylococcus aureus.
C. eikenella corrodens.
D. fusobacterium nucleatum.

A

Answer: A

Rationale:
The human and animal population is very similar in their oral flora. Each has a predomination of Streptococcus species and numerous gram negative anaerobic organisms. Each has numerous species of viruses and some mycobacteria have been reported. It is however, Pasteurella multocida that is only found in the non-human population

Reference:
Haug RH, Assael L, “Infections in the Maxillofacial Trauma Patient”, Chapter 17 in Topazian RG, Goldberg MH, Hupp J (Eds.). Oral and Maxillofacial Infections. Philadelphia PA, W B Saunders, 2002, pp 359-380.

108
Q
What is the most commonly fractured area of the mandible in pediatric patients?
A. Symphysis
B. Parasymphysis
C. Coronoid process
D. Condyle
A

Rationale:
Condyle fractures have been reported to occur most commonly in pediatric patients (15-
60%). With increasing age, the angle is involved more commonly.
Anatomic Distribution of Mandibular Injury in Pediatric Patient
Condyle 15-60%
Alveolus 8-50.6%
Body 6-44%
Symphysis 2-40%
ParaSymph 23-33%
Angle 3-27%
Ramus 1-10%
Coronoid 0-19%
Reference:
Kaban LB, et. al. Pediatric Oral and Maxillofacial Surgery. Ch. 25 Facial Trauma II:
Dentoalveolar Injuries and Mandibular Fractures, Baumann A, et. al. p.446.
Haug R, Foss J. Maxillofacial injuries in the pediatric patient. Oral Surgery, Oral
Medicine, Oral Pathology, Oral Radiology & Endodontics, 90:2, 2000, p.131.

109
Q
What explains the relative paucity of pediatric midfacial trauma compared to adults?
A. Large paranasal sinuses
B. Rigid facial bones
C. Prominent midfacial structures
D. Size of cranium
A

Answer: D
Rationale:
The low incidence of midfacial fractures in children can be explained by the elasticity of
the child’s facial bones, the retrusive position of the maxilla, nose and infraorbital rims and
the anatomic protection afforded by the cranium.
Reference:
Kaban LB, et. al. Pediatric Oral and Maxillofacial Surgery. Ch. 24 Facial Trauma I:
Midfacial Fractures, Baumann A, et. al. p.425.
Posnick JC. Craniofacial and Maxillofacial Surgery in Children and Young Adults. Ch 30
Primary Craniomaxillofacial Fracture Management, Posnick JC. p.704, 716.

110
Q

Which of the following is a consideration for open reduction and internal fixation in pediatric
patients with condyle fractures?
A. Minor malocclusion in the mixed dentition
B. Minor malocclusion in the primary dentition
C. Persistent malocclusion in the permanent dentition
D. To restore vertical height of the ramus regardless of age or occlusion

A

Answer: C
Rationale:
Open reduction internal fixation (ORIF) is rarely indicated in the pediatric population.
Most condylar fractures can be treated with a combination of intermaxillary fixation or
analgesics and soft diet. Minor occlusal discrepancies will equilibrate with continued
growth and dental eruption. Children in the permanent dentition who have persistent
malocclusion at the time of initial treatment or after a period of intermaxillary fixation
should be considered for ORIF to restore ramal height and occlusal function.

111
Q

How do pediatric hemorrhagic shock and circulatory decompensation differ from the adult?
A. There are superior compensatory mechanisms in pediatric patients
B. Hemorrhagic shock occurs gradually in children despite rapid blood loss
C. Hemorrhagic shock occurs more suddenly, as vasoconstriction, tachycardia, and
myocardial contractility can mask signs and symptoms
D. Hemorrhagic shock rarely occurs in the pediatric population because of a higher body
surface area to mass ratio

A

Answer: C
Rationale:
Management of the pediatric trauma patient differs from that of the adult. Lack of
compensatory reserves in the pediatric patients can lead to a sudden cardiovascular
collapse. This occurs rather quickly since tachycardia, vasoconstriction and increased
myocardial contractility will initially mask the presenting signs and symptoms. Body
surface ratio does not affect shock response, but does affect both heat and insensible fluid
loss in the pediatric patient.
Reference:
Haug R, Foss J. Maxillofacial injuries in the pediatric patient. Oral Surgery, Oral
Medicine, Oral Pathology, Oral Radiology & Endodontics, 90:2, 2000, p.128.
Miloro M, et. al. Peterson’s Principles of Oral and Maxillofacial Surgery. Ch 27 Pediatric
Craniomaxillofacial Fracture Management, Posnick JC, et. al. p.528.

112
Q

A trauma patient has multiple facial lacerations heavily contaminated with soil. He has
completed a tetanus immunization series but has not received a booster for the past 7 years. How
should this patient be managed?
A. Tetanus toxoid booster injection
B. Tetanus booster and tetanus immunoglobulin
C Tetanus immunoglobulin
D. Tetanus booster, tetanus immunoglobulin plus full course of immunization

A

Answer: A
Rationale:
In a “clean” (minimally or noncontaminated) would in a previously immunized patient who
has not received a booster in the past 10 years, a booster dose of 0.5 ml of toxoid booster is
recommended. This timeline decreases to 5 years for a “dirty” or contaminated wound. If
there is no history of immunization or an unknown history, the tetanus immunization series
is begun; with the addition of tetanus immune globulin (TIG) for dirty wounds.

113
Q

Indication for removal of teeth in the line of mandibular fractures includes which of the
following?
A. Complete bony impacted wisdom teeth in the line of fracture
B. Teeth with exposed root apices in the fracture site
C. Teeth with coronal fractures
D. Immobile teeth in line of fracture

A

Answer: B
Rationale:
Relative indications for removal of teeth in the line of mandibular fractures include:
Presence of obvious pathology (caries, periodontal disease)
Gross mobility of involved teeth
Teeth that prevent adequate reduction of fracture
Teeth with fractured roots
Teeth with exposed apices
Reference:
Peterson 2nd edition, Principles of management of mandibular fractures. P 401-434,
Fonseca, Oral and Maxillofacial Trauma, Mandibular fractures, P 479-522, 2005,

114
Q

Which of the following is a relative contraindication for closed reduction of a mandible fracture?
A. Poorly controlled seizure disorder
B. Poorly controlled renal disease
C. Well-controlled psychiatric disorder
D. Well-controlled chronic obstructive pulmonary disease

A

Answer: A
Rationale:
Poorly controlled seizure disorders, mental retardation, and psychiatric disorders are
relative contraindication for closed reduction of mandible fractures in which
maxillomandibular fixation would be poorly tolerated. . If possible, an attempt should be
made to perform an open reduction with rigid fixation for these patients.
Reference:
Peterson 2nd edition, Principles of management of mandibular fractures. P 401-434, 2004
Fonseca, Oral and Maxillofacial Trauma, Mandibular fractures, P 479-522, 2005

115
Q
What is the recommended treatment for a patient with an intracapsular condylar head fracture
and normal occlusion?
A. Observation
B. 2 weeks of intermaxially fixation
C. 6 weeks of intermaxially fixation
D. 8 weeks of intermaxillary fixation
A

Answer: A
Rationale:
Intracapsular condylar head fractures with a normal occlusion should be observed. These
patients do not need intermaxillary fixation since there is no change in occlusion. The
management also includes soft dies, pain control and range of motion exercise to regain
normal interincisal opening after the initial healing period.
Reference:
Peterson 2nd edition, Principles of management of mandibular fractures. P 401-434, 2004
Fonseca, Oral and Maxillofacial Trauma, Mandibular fractures, P 479-522, 2005

116
Q

Which of the following is a contraindication for exploration of an orbital floor fracture?
A. Enophthalmos
B. Grade III Hyphema
C. Persistant diplopia
D. Limitation of extraocular muscle function

A

Answer: B
Rationale:
Hyphema is a contraindication to orbital exploration due to the possibility of re-bleeding
into the anterior chamber, which can lead to corneal staining and glaucoma.
Reference:
Peterson 2nd edition, Principles of Oral & Maxillofacial Surgery, Orbital and ocular
trauma, P 463-490, 2004
Brandt MT, Haug RH. Traumatic hyphema: a comprehensive review. J Oral Maxillofac
Surg, 59, 1462, 2001.

117
Q
Which of the following is least likely to cause diplopia?
A. Chemosis
B. Hematoma
C. Extraocular muscle entrapment
D. Abducens nerve injury
A

Answer: A
Rationale:
Chemosis is edema of palpebral and bulbar conjuctiva. This does not lead to diplopia.
Other conditions including hematoma, muscle entrapment and a nerve injury will cause
double vision.
Reference:
Peterson 2nd edition, Principles of Oral and Maxillofacial Surgery, Orbital and ocular
trauma, P 463-490, 2004
Fonseca, Oral and Maxillofacial Trauma, ophthalmic consequences of maxillofacial
trauma, p 693-720, 2005

118
Q

Which of the following is the clearest indication for open reduction of a mandibular condyle
fracture?
A. Unilateral condylar fractures with comminuted upper face fracture
B. Displacement into middle cranial fossa
C. Medial extracapsular dislocation of the condylar head
D. Isolated condylar neck fracture

A

Answer: B
Rationale:
Zide’s criteria for open reduction of condyle fractures includes the following relative and
absolute indications:

Reference:
Zide MF. Open reduction of mandibular condyle fractures. Indications and technique. Clin
Plast Surg. 1989 Jan; 16(1):69-76.
Peterson 2nd edition, , Principles of management of mandibular fractures. P 401-434,
2004
Fonseca, Oral and Maxillofacial Trauma, Mandibular fractures, P 479-522, 2005

119
Q

A patient has a deep, vertical laceration on his face extending from the mid lower lid to the
ipsilateral oral commissure. Physical examination reveals weakness of the buccal branches of
the facial nerve. How should this patient be managed?
A. Explore the wound to identify and repair the injured nerve
B. Irrigate and close the wound without nerve exploration
C. Repair the laceration and wait two weeks for return of function, if no improvement in
function, then explore and repair
D. Consider nerve grafting from the great auricular nerve

A

Answer: B
Rationale:
Facial nerve injuries medial to a vertical line from the lateral canthus do not need to be
explored because there is increased terminal facial nerve branch arborization and therefore
multiple small branches exist. Exploration of such small branches is not practical. This
wound should be managed as any other facial laceration.
Reference:
Peterson 2nd edition, Principles of Oral & Maxillofacial Surgery, Soft tissue injuires, P
357-370, 2004
Fonseca, Oral and Maxillofacial Trauma, Applied surgical anatomy of the head and neck, p
281-328, 2005,

120
Q

Following repair of pan-facial fractures, a patient develops diplopia, orbital proptosis, and
reports an unusual buzzing sound. What is the most likely diagnosis?
A. Cavernous sinus thrombosis
B. Orbital apex syndrome
C. Carotid-cavernous fistula
D. Temporal arteritis

A

Answer: C
Rationale:
Carotid-cavernous fistulae (CCF) are potential complications of craniofacial trauma. Signs
and symptoms are related to the abnormal flow of blood (shunting) between a high flow
(internal carotid) to a low flow system (cavernous sinus). Patients often complain of a
swollen red eye, orbital pain, loud buzzing sound and swishing sound, proptosis and visual
changes. Although cavernous sinus thrombosis, orbital apex syndrome and temporal
arteritis may have visual disturbances, none will display an unusual buzzing sound. CCF
can occur within days to weeks following trauma.
Reference:
Origitano TC, Al-Meffy O. Aneurysms of the cavernous sinus: treatment options and
consideration. In: Youmans RJ. Neurological Surgery. Philadelphia: WB Saunders,
1995, 1320-1335.
Fattahi T, Brandt MT, Jenkins WS, et al. Traumatic carotid-cavernous fistula:
pathophysiology and treatment. J Craniofacial Surg, 14:1, 2003.

121
Q
Injury to which zone of the neck is most amenable to physical examination?
A. Zone 1
B. Zone 2
C. Zone 3
D. Zone 4
A

Answer: B
Rationale:
Penetrating neck injuries can have significant morbidity and mortality. Zone 1 extends
from the clavicle to the cricoid. Zone 2 is from the cricoid to the inferior border of the
mandible. Zone 3 is from the mandibular angle to the skull base. Physical examination of
zone 2 is most easily accomplished due to its location and ease of access. While studies in
addition to physical exam may be indicated in Zone 2 penetrating injuries, Zones 1 and 3
more often require further diagnostic tools such as an arteriogram or esophagoscopy.
Reference:
Advanced Trauma Life Support Student Manual. Initial assessment and management, P
21-46, American College of Surgeons. Sixth Edition. 1997.
Miloro, M. et al, Peterson’s Principles of Oral and Maxillofacial Surgery, Second Edition,
Initial management of the trauma patient, p 327-356, BC Decker 2004, chapter 18

122
Q
What is the most likely position of a displaced mandibular condylar fracture?
A. Superior and lateral
B. Superior and medial
C. Anterior and medial
D. Anterior and lateral
A

Answer: C
Rationale:
Condylar displacement usually occurs in an anterior and medial direction secondary to the
lateral ptyergoid muscle pull. The patient will have a premature bite on the ipsilateral side
and upon opening will deviate to the ipsilateral side as well.
Reference:
Peterson 2nd edition, Principles of Oral and Maxillofacial Surgery, Principles of
management of mandibular fracture, p 401-434, 2004
Fonseca, Oral and Maxillofacial Trauma, Mandibular fractures, p 479-522, 2005

123
Q
Formation of a cartilaginous callus following an initial formation of a hematoma marks what
type of bone healing?
A. Initial
B. Primary
C. Secondary
D. Tertiary
A

Answer: C
Rationale:
Bone healing has been divided into 2 categories: primary and secondary. Primary bone
healing is characterized by less than 1 mm gap between bone edges, no bony mobility and
no callus formation. This type of healing occurs following placement of load-bearing
plates. Secondary bone healing occurs with formation of a hematoma, fibroblast
infiltration, cartilaginous and bony callus formation, and then remodeling. This type of
healing occurs following closed reduction of fractures.
Reference:
Peterson 2nd edition, Principles of Oral and Maxillofacial Surgery, Principles of
management of mandibular fracture, p 401-434, 2004
Fonseca, Oral and Maxillofacial Trauma, Mandibular fractures, p 479-522, 2005
Abubaker OA. Oral and Maxillofacial Surgery Secrets. Maxillofacial trauma, 197-206
2001

124
Q

A 25 year-old female presents to the emergency department with a medial pole fracture of the right TMJ condyle. Treatment should consist of which of the following?
A. Open reduction and internal fixation followed by maxillomandibular fixation for 14 days
B. Open reduction with internal fixation only
C. Closed reduction with maxillomandibular fixation for 4 weeks to maintain proper posterior vertical height.
D. Medical management with soft diet only

A

Answer: D
Rationale:
The medial pole of the TMJ condyle by position does not function as a primary determinant of mandibular vertical height. As such, open or closed surgical approaches for such a fracture are unwarranted; and the injury is managed with soft diet and range of motion exercises.
Reference:
Sicher, H; and DuBrul, EL: Temporomandibular articulation. In, Oral Anatomy (6th Ed.) CV Mosby Co. St. Louis. pp. 160 - 191, 1975.
Quinn, PD: Color Atlas of Temporomandibular Joint Surgery. CV Mosby, St. Louis, 1998.
Assael, LA: Hard Tissue Trauma. In, Temporomandibular Joint Disorders - Diagnosis and Treatment. Kaplan, AS and Assael, LA (Eds) WB Saunders, Philadelphia, 1992. pp.224-237.
Mercuri, LG: Temporomandibular Joint Disorders. In: Kwon, PH; and Laskin, DM (eds) Clinician’ Manual of Oral and Maxillofacial Surgery 3rd ed. 2001 Quintessence. Chicago.
Kobinson DA, Hohn FI: Traumatic injuries. In, Laskin DM, Greene CS, Hylander WL (eds): Temporomandibular Disorders: An Evidence-Based Approach to Diagnosis and Treatment. 2006 Quintessence. Chicago.